Sei sulla pagina 1di 56

Facial plaques in aging men

CASE #2: Tumid lupus erythematosus


Biopsy revealed both superficial and deep lymphohistiocytic inflammatory infiltrates
as well as dermal mucin with little or no epidermal or dermo-epidermal involvement,
and a diagnosis of tumid lupus erythematosus (TLE) was made.
A rare subtype of chronic cutaneous lupus erythematosus, TLE manifests as a
succulent, edematous, and persistent plaque, most commonly on the face. The
presentation is similar to that of polymorphous light eruption, the lymphocytic
infiltration of Jessner, systemic lupus erythematosus (SLE), reticulated erythematous
mucinosis, or gyrate erythema.
Thought to be a cutaneous manifestation of lupus erythematosus, TLE sometimes
occurs in isolation and is probably not associated with SLE. Histopathologically, TLE
and SLE are similar, but TLE patients are usually negative for antinuclear antibodies,
while SLE patients are positive. Effective treatment regimens include topical and oral
steroids, chloroquine (antimalarials), and sunscreen.

TLE following highly


active antiretroviral therapy for HIV infection is thought to be a manifestation of
immune restoration. A case of blaschkolinear cutaneous tumid lupus erythematosus
has been reported.

Our patient's laboratory studies, including an antinuclear antibody titer, were normal.
He is currently being treated with topical clobetasol.

CASE #1: Exanthematous drug eruption


Most drug eruptions manifest with erythematous macules and papules, giving them an
appearance that may be referred to as exanthematous, mobilliform, or scarlatiniform.
The basis for this is not clear. We will refer to them as exanthematous drug eruptions
(EDEs)
EDEs can emerge as soon as 48 hours after a medication is administered. Onset is
often within the first two weeks of administration, although some EDEs manifest as
much as 10 days after a medication has been stopped. EDEs usually are not associated
with fever or systemic changes, such as lymphadenopathy. Drug reactions associated
with lymphadenopathy are referred to as drug eruption with eosinophils and systemic
symptoms (DRESS) or pseudolymphoma. A genetic basis for such severe drug
reactions as DRESS and pseudolymphoma continues to be defined.

EDEs typically do not


manifest on the face. Instead they tend to start on the central body and spread
centrifugally. In my experience, the rash often starts on the back of hospitalized
patients and in the axilla or groin of others, progressing next to the front of the body
and then to the arms and legs. The eruption can be confluent. Some EDEs occur only
in sun-exposed areas, the so-called UV recall reaction. A sunburn as many as two
weeks to 10 months prior to development of the reaction may have provided a fecund
setting for such a reaction.
The chief differential diagnoses of EDE include viral eruptions and insect bites. Most
viral eruptions do not itch, while bites and some EDEs are associated with pruritus.
Generally the rash of insect bites is not so generalized or confluent as that of an EDE.
Histology can distinguish the three diagnoses and help explain the absence or
presence of pruritus. The typical EDE involves an interstitial and perivascular
infiltrate of lymphocytes and often eosinophils. The infiltrate is usually superficial but
can be both superficial and deep. Histologically, viral eruptions manifest only with
lymphocytes. Because eosinophils and mast cells are key to the development of
pruritus, eruptions that have only lymphocytes do not itch. Clinically, bites have a less

uniform appearance, and histologically, they have a deeper and more wedge-shaped
array of superficial and deep eosinophils.
EDEs are a type IV hypersensitivity reaction. Hypersensitivity reactions are divided
into four categories: Type I hypersensitivity reactions are allergic and immediate and
involve immunoglobulin E (IgE)-dependent reactions that can result in hives and
breathing problems, while type II hypersensitivity reactions involve cytotoxic T-cells
that lyse other cells and can result in hemolysis and purpura. Like type III
hypersensitivity reactions, type IV hypersensitivity is a delayed-type reaction
requiring the formation of haptens, which are immune complexes composed of
immunoglobulin and molecules of the allergen. In type IV hypersensitivity,
lymphocytes carry the haptens back to the lymph nodes and return to the site of the
reaction sensitized to the allergen. Drug-induced hypersensitivity syndrome and
allergic contact dermatitis are other examples of type IV hypersensitivity reactions.
Some medications are more likely to cause EDEs than others. The medications most
commonly involved are sulfa antibiotics and amoxicillin/ampicillin. Aromatic
anticonvulsants, such as phenytoin (Dilantin), also cause allergic reactions. Other
possibilities include allopurinol (Zyloprim) and abacavir (Ziagen). Bear in mind that
any medication can cause a reaction at anytime. Moreover, patients can react to a
combination of medications even if they haven't previously reacted to an individual
medication. Use of ampicillin/amoxicillin in certain disorders seems to carry a higher
risk of EDE. For example, 95% of patients who are infected with Epstein-Barr virus
and 30% of patients with chronic lymphocytic leukemia may experience EDE when
treated with ampicillin/amoxicillin. Pregnancy that leads to a recrudescence of EBV
infection can cause patients to have develop an EDE following use of ampicillin, a
popular antibiotic among obstetricians.
When an EDE is suspected, make a list of all the patient's medications and when they
were started (and stopped, if appropriate). Discontinue all unnecessary medications,
using particular care when stopping psychiatric agents.
Treatment of EDE is primarily symptomatic and supportive. Begin by trying to
identify the offending medication and stopping it. Apply triamcinolone 0.1% ointment
(available in a 1-lb size) to all affected areas. Side effects, such as adrenalsuppression,
are much less likely with a class IV corticosteroid than with a class I corticosteroid,
such as clobestasol (Temovate), and patients will have sufficient steroid to cover their
entire body.
An antihistamine such as hydroxyzine 25 mg used at night can aid with sleeping;
fexofenadine (Allegra) 180 mg can sometimes be used in the daytime. Eruptions tend
to clear within two weeks of discontinuing the offending agent and starting therapy.
The patient knows that the eruption is resolving when areas of erythema evolve into
areas of desquamation.
This patient's ampicillin was stopped, and he was treated with triamcinolone 0.1%
ointment and hydroxyzine 25 mg at night. Within a week, the erythema evolved into
desquamation. At no time did the patient have a fever or abnormal laboratory value

pair of generalized red rashes

CASE #2: Telangiectasia macularis eruptiva perstans


Also referred to as paucicellular mastocytosis, telangiectasia macularis eruptive
perstans (TMEP) is just one of several forms of cutaneous mastocytosis. The other

variants include urticaria pigmentosa, maculopapular cutaneous mastocytosis, diffuse


cutaneous mastocytosis, mastocytoma of the skin, and erythrodermic mastocytosis.
Cutaneous mastocytosis is an uncommon cause of skin disease.
The epidemiology of TMEP has yet to be fully defined. While urticaria pigmentosa,
the most common of the cutaneous mastocytoses, is common in children, TMEP is
very rare in children. Familial TMEP has been reported and can occur in children. In
the United States, 0.1%-0.8% of new patients seeking care in dermatology clinics
have some form of mastocytosis. Less than 1% of patients visiting dermatology
clinics with mastocytosis have TMEP.

The physical
manifestations of TMEP include telangiectases measuring 2-6 mm, sometimes on a
background of erythema or tan discoloration. TMEP represents permanent
vasodilation of the skin, secondary to mast-cell release of chemical mediators and
angiogenic factors. TMEP most commonly involves the trunk but has been reported in
rare instances to occur unilaterally. Heat and pressure may produce localized urticaria.
TMEP rarely has systemic complications. Most patients complain only of variable
degrees of pruritus. Darier's sign, a wheal surrounded by erythema in an area of skin
with mast cells, is almost always present in urticaria pigmentosa but may not be
present in TMEP. Rare cases of TMEP are associated with episodic headaches,
flushing, GI upset, palpitations, syncope, hepatosplenomegaly, increased numbers of
mast cells in the bone marrow, and abnormal findings on skeletal radiographs.
The differential diagnosis of TMEP includes essential generalized telangiectases
(those simply present since birth), nevoid telangiectases, telangiectases due to liver
disease, carcinoid syndrome rosacea, carcinoid syndrome, hepatopathy or
collagenosis, and hereditary hemorrhagic telangiectasia. TMEP manifesting as a
pseudoallergic food reaction has been reported.1 One woman presented late in the
second trimester with an anaphylactoid reaction, rash of blood vessels, uterine
contractions, and vaginal bleeding.2 The patient had a markedly elevated urinary
histamine level, and skin biopsy revealed perivascular mast-cell infiltration,
establishing the diagnosis of TMEP. She was treated successfully with tocolytics and
antihistamines.
The histologic findings of TMEP involve mast cells surrounding capillary venules and
the superficial venular plexus. Mast cells have a fried egg appearance and can be
definitively detected with Giemsa, toluidine blue, chloroacetate esterase, and

aminocaproate esterase stains, which are picked up by metachromatic cytoplasmic


granules. On occasion, the mast cells are somewhat sparse, complicating the pure
histologic establishment of a diagnosis and necessitating clinicopathologic
correlation.
TMEP associated with malignant melanoma was reported in 2009, but no role for
TMEP as a marker of malignancy has yet been established. Mutations in the c-kit
(mast cell growth factor) proto-oncogene are not found in all patients with cutaneous
mastocystosis. Alterations in this proto-oncogene may indicate more aggressive
mastocytosis and have been observed in melanoma but not TMEP.
Although TMEP is a cutaneous manifestation of mastocytosis, systemic involvement
may occur. Elevated serum tryptase levels may serve as a guide for systemic
involvement. In a study of 52 patients, the prevalence of bone-marrow biopsy
specimens indicating systemic mastocytosis rose as the level of total tryptase
increased. The biopsy specimens were 100% positive when total tryptase was >75
ng/mL and 50% positive when total tryptase levels were 20-75 ng/mL.3
Treatment of TMEP, if it is symptomatic, involves H1- receptor antagonists, such as
hydroxyzine, to help reduce pruritus or flushing. The 585-nm flashlamp-pumped dye
laser, which targets blood vessels, has yielded some success. Pre-laser treatment with
appropriate H1- and H2-receptor blockers is important to avoid potential
complications from laser-induced mediator release. Total electron beam radiation has
been successfully used in one patient.4 Psoralen with UVA and UVB, which inhibits
the release of histamine by mast cells, has been used for the treatment of
mastocytosis. This approach might also work for TMEP, but no studies have been
reported.
Workup of this patient, who had few symptoms, revealed laboratory results that were
within normal limits. He was reassured that his skin eruption, while noticeable, did
not pose a threat to his overall health.
Dr. Scheinfeld is assistant clinical professor of dermatology at Columbia University
in New York City, where he has a private practice.

A pair of patients with purple lesions


CASE #1: Dermatofibroma
Usually seen on the legs of young women, dermatofibromas (DFs) manifest as purple
papules that dimple inward when squeezed (the dimple sign). Whether DFs are
neoplasms or a reactive process remains a matter of debate, but the evidence suggests
that they are benign neoplasms. The proximate etiology of most DFs seems to be
insect bite or skin trauma secondary to shaving. Also known as fibrous histiocytomas
or sclerosing angiomas, DFs are among the lesions most commonly seen in
dermatology practices.
DFs occur more often in women than in men by a ratio of 4 to 1. While the patients
most commonly affected appear to be women between the ages of 20 and 40 years, an
estimated 20% of DFs occur in persons younger than 20.
While DFs are usually purple, they can manifest in a range of colors, including blue,
brown, gray, orange, pink, mauve, red, yellow, or black as well as combinations
thereof. DFs typically range in size from 0.3 cm to 1.5 cm, with 0.5-0.7 cm being the
most common size. DFs can be ulcerated or eroded. Variants include giant (>5 cm in
diameter), atrophic, and atypical polypoid DF, as well as DF with satellitosis. Clinical

diagnosis is made by squeezing to elicit a dimple sign or by dermoscopy revealing a


peripheral pigmentary network with a central white area.

Most commonly seen


on the legs, DFs can occur on the arms, hands, and torso. They are rarely found on the
face. DF-like lesions on areas other than the legs or arms should prompt consideration
of other diagnoses.
Histologically, DFs are formed from dermal dendritic histiocyte cells. The overlying
epidermis demonstrates acanthosis. Pseudoepitheliomatous hyperplasia and
proliferation of basal cells can occur. Increased pigment, likely iron or melanin, can
be observed. Most lesions display a grenz zone of normal papillary dermis overlying
the fibrous cells. DFs usually demonstrate whorling fascicles of a spindle cell
proliferation, with excessive collagen deposition wrapping around normal collagen
bundles at the periphery.
Immunohistochemical testing of most DFs reveals antibodies to factor XIIIa.
Transforming growth factor- signaling may underlie the fibrosis of DF.
Histopathologic variants are numerous and include cellular, sclerotic fibromalike
(different from a sclerotic fibroma), aneurysmal, epithelioid, atypical, palisading, and
cholesterotic DF; on the hands and feet, myxoid and schwannomalike variants have
been noted.
Multiple DF is defined as manifestation of 15 or more DFs. Conditions associated
with multiple DFs include lupus and HIV.1 Eruptive DFs is the term used to describe
the sudden development of DFs. Eruputive DFs have been associated with
immunosuppression from drugs, e.g., efalizumab; neoplasms, such as Szary
syndrome; multiple IgA myeloma; chronic myelogenous leukemia; and diseases
involving immunosuppression, e.g., HIV and lupus.2
The differential diagnoses for DF include keloid, basal cell carcinoma, blue nevi,
prurigo nodularis, Spitz nevi, sclerotic fibroma, multinucleate cell angiohistiocytoma,
and pilomatricoma. The painful tumors of skin can also imitate DF; these include blue
rubber bleb nevus, leiomyoma, eccrine spiradenoma, neuroma, DF, angiolipoma,
neurilemoma, endometrioma, glomangioma, and granular cell tumor.
Dermatofibrosarcoma protuberans (DFSP), which is usually factor XIIIa-negative and
CD34-positive, is also on the list of differential diagnoses. A slow-growing but
insidious neoplasm, DFSP may be the most important condition requiring rule out.

DFs not associated with any symptoms can be left untreated. The most frequently
reported symptoms are pruritus and tenderness. DFs are likely the most common of all
painful tumors. For symptomatic DF, treatment with liquid nitrogen can be attempted;
if several such treatments fail, intralesional steroid injections (2-3 mg/cc) or shaving
the lesions can be attempted. Surgery on the lower legs of young women often results
in a suboptimal cosmetic result and should be considered carefully before being
attempted. Ablative laser treatment has been reported but is rarely used.
Because the patient was not bothered by the lesion, we decided to provide reassurance
and not pursue any additional therapy.

A pair of patients with purple lesions

CASE #2: Isolated lichen planus of the penis


The characteristics of lichen planus (LP) can be recalled by remembering the Ps:
purple, polygonal, polymorphous, pruritic, and papular with polished surfaces. LP is
seen most often on the wrists and arms but is not uncommon on the genitals and
mucous membranes (where it looks white). In the mouth, the white manifestations of
LP can appear reticulated and are referred to as Wickham's striae.
LP can be diagnosed clinically, but biopsy is helpful. In erosive disease, even if the
clinical impression is of Wickham's striae, mouth lesions should be biopsied,
particularly in smokers. In bullous LP, the clinical impression is usually of such
diseases as bullous pemphigoid or pemphigus vulgaris, and only histologic
examination reveals the true diagnosis.
Pathologically, lymphocytes attack keratinocytes and lead to an interface dermatitis.
Degenerative keratinocytes, referred to as colloid or Civatte bodies, manifest in the
lower epidermis. Direct staining by immunofluorescent techniques may reveal
immunoglobulin deposits at the base of the epidermis. While the cause of LP is not
known, some immune basis seems likely, and haplotype B7 is more common in
patients with LP. Some association between LP and hepatitis C has been observed,
particularly in Japan. Therefore viral induction of the pathologic process of LP is
possible.

LP has many variants.


Hypertrophic LP often appears on the shins and can resemble prurigo nodularis. In
men, erosive LP has a greater likelihood of inducing cancer. LP of the mucous
membranes is present in about 50% of cases. LP follicularis, which involves the scalp,

can lead to a scarring alopecia that manifests with small red spiny papules around a
cluster of hairs. Atrophic LP looks more white than purple. LP pigmentosus is more
common in darker-skinned individuals and can lead to brown and purple
discoloration. LP pemphigoides is an overlap of LP and bullous pemphigoid. Linear
LP must be distinguished from lichen striatus, linear psoriasis, and linear
porokeratosis. LP of the nail is present in about 10% of cases and can cause pitting,
grooving, and ridging of the nails. There can also be onycholysis. LP of the nails
overlaps with 20-nail dystrophy (which can also be the result of alopecia areata) and
can lead to scarring of the nail, including the formation of a pterygium, a scarring
process on the underside of the nail. Vaginal LP can lead to scarring of the vagina,
impairing sexual activity. Other variants include actinic, annular, and bullous LP.
The onset of LP can be sudden or gradual. Its incidence is the same in both sexes. LP
is most common in middle age (30-70 years) but can occur at any age.3 Most cases of
LP will resolve, unlike diseases such as psoriasis or lupus that can linger for a
lifetime. The differential diagnosis of LP includes bullous diseases, dermatofibromas,
lichen sclerosus, graft-versus-host disease, psoriasis, lichen nitidus, prurigo nodularis,
syphilis, pityriasis rosea, and tinea. Drugs can cause a rash that resembles LP; this is
called a lichenoid drug eruption. The most common causes of lichenoid drug
eruptions are gold, antimalarial agents, and captopril. Actinic lichenoid drug
eruptions, which occur in sun-exposed areas, are caused by antimalarials and thiazide
diuretics.
Treatment of limited LP comprises topical steroids.4 Extensive LP can be treated with
oral steroids, UV phototherapy, and oral acitretin. Methotrexate and
hydroxychloroquine are also used for extensive cases of LP. Topical calcineurin
inhibitors can be used to treat oral and erosive LP. Lichen planopilaris, which occurs
in the scalp, is commonly treated with injection of corticosteroids.
In this case, a clinical diagnosis of LP was made; the patient declined biopsy of the
lesion. After four weeks of treatment with triamcinolone 0.1% cream, the lesion had
resolved, leaving behind some pigmentary alteration.

Acne: Choosing the best treatments


Key Points

Acne severity correlates with increased anxiety, anger, and other negative
psychological consequences.
Menstruation, pregnancy, and hyperandrogenic states can exacerbate
acne.
Retinoids and retinoid-analogs prevent development of the microcomedo
and comprise reasonable first-line agents.
A topical retinoid combined with an antibiotic (either topical or oral)
yields greater and faster results than either used alone.

More than 17 million Americans are affected by acne vulgaris every year.1 A disease
predominantly of adolescence and young adulthood, acne vulgaris afflicts 85% of
Americans between the ages of 12 and 24 years.2 The average age of onset is 11 years
for girls and 12 years for boys.3 Although primarily a disease of teenagers,
approximately 12% of women and 3% of men continue to have acne until age 44.4

Racial differences exist: The mean age of onset is 15.9 years in Hispanics, 18.9 years
in Asians, and 20.3 years in African Americans. Hispanic teenagers not only have the
highest prevalence of acne, they also develop more scarring and are more likely to
have severe nodular acne.5 African-American patients with acne are often more
concerned with the postinflammatory hyperpigmentation caused by the lesions than
they are with the active lesions themselves.6
Psychosocial impact
Because acne vulgaris has a predilection for the face and affects adolescents, it would
be expected to have a significant impact on an individual's quality of life (QOL). In
the past, generalized instruments (not specific to skin disease) were used to measure
the QOL in patients with skin disease.7-9 Such studies found that acne severity
correlated with increased anxiety, anger, and other negative psychological
consequences. The Dermatology Life Quality Index (DLQI) provides global QOL
assessment specific for skin disease.1 The initial study found that acne patients had
statistically higher scores on the DLQI, indicating a decrease in QOL.10 More recently,
depression and suicidal ideation were studied in 480 potentially disfiguring
dermatologic conditions.11 Depression and suicidal ideation were more common in
those with acne than in patients with alopecia areata and mild-to-moderate psoriasis.
Moreover, psychiatric disorders can develop secondary to acne.11
Pathophysiology
The first step in the formation of acne is the development of the microcomedo.12
Normally, keratinocytes that line the follicular opening are shed into the lumen of the
pilosebaceous unit and excreted. Comedone formation occurs when increased
cohesiveness prevents the shedding of keratinocytes, and they accumulate in the
opening, leading to hyperkeratosis. Increased production of follicular keratinocytes
also occurs. These cells, along with lipids and other cellular elements, create a plug in
the follicular opening. With time, lipids, cellular debris, and bacteria
(Propionibacterium acnes) accumulate behind the comedone and form a clinically
apparent lesion.

Open comedones
(blackheads) are flat or dome-shaped papules with a dilated follicular orifice filled
with black-colored (oxidized) keratin. Closed comedones (whiteheads) are more
subtle and present as small, pale or flesh-colored papules with no apparent follicular
orifice or erythema. Comedones may become secondarily inflamed if squeezed,
picked, or scratched. Inflammatory lesions include erythematous papules, pustules, or
nodules (Figure 1). Acne vulgaris occurs primarily on the face and, to a lesser degree,
on the upper chest, back, and shoulders.

Hormones can influence the course of acne. In one study, almost half of adult females
had premenstrual flares of acne.13 Many women experience their first flare of acne or
worsening of existent acne with pregnancy. Conditions in which patients are exposed
to excess androgens, whether endogenous or exogenous, are conducive to the
development of acne.
Diagnosis

The clinical
presentation of acne vulgaris typically renders a straightforward diagnosis. The age of
the patient and the morphology and distribution of lesions are often characteristic.
Severity can range from mild to moderate (Figure 2) to severe (Figure 3). Rarely, a
biopsy may be required for a firm diagnosis.
The differential diagnosis for acne is quite broad. Acne rosacea can be similar to acne
vulgaris with papulopustules and erythema on the face, especially the nose and
cheeks. Perioral dermatitis is characterized by small papules and vesiculopustules on
an erythematous base with scale that closely follows the border of the lips and,
occasionally, the paranasal and periorbital regions.

Gram-negative
folliculitis is characterized by numerous superficial pustules or deep-seated nodules.
Pseudofolliculitis barbae should be considered in African-American patients,
especially men presenting with inflammatory papules and/or pustules of the beard
area (Figure 4). Acne keloidalis nuchae, most common in African Americans, presents
with persistent inflammatory papules and pustules over the occipital scalp and
posterior neck. In patients with HIV/AIDS, eosinophilic pustular folliculitis should be
considered in the differential diagnosis of acne. It presents as recurrent, pruritic,
follicular papules and pustules on the face, neck, trunk, and proximal extremities.
Several disorders of follicular keratinization exist and can present on the face.
Keratosis pilaris on the face can mimic acne vulgaris. Keratosis pilaris atrophicans
faciei is characterized by erythema and small, horny, follicular plugs on the cheeks,
temples, and forehead.

Acneiform eruptions
can be caused by many medications, most commonly corticosteroids, anabolic
steroids, isoniazid, phenytoin, lithium, bromides, and iodides. Generally, this eruption
is monomorphous and consists of inflammatory papules and pustules. Acne

cosmetica, the result of comedogenic cosmetics, presents with closed comedones and
papulopustules mainly on the cheeks and chin. Discontinuing the offending agent
usually allows resolution. Chloracne, caused by chlorinated aromatic hydrocarbons,
presents with small cystic papules and nodules on the malar, retroauricular, and
mandibular regions of the head and neck.
Acne mechanica results from mechanical forces (rubbing) causing occlusion of the
pilosebaceous orifice. Helmets, chin straps, violins, and collars are among the
implicated objects. Acne excorie refers to patients with or without mild acne who
scratch and pick their faces and leave small eroded, crusted lesions.

Acne: Choosing the best treatments


Treatment
Successful treatment of acne must begin with patient education. Initial patient
encounters should focus on realistic goals of therapy while emphasizing the chronic
nature of the disease. Proper skin hygiene is crucial for acne patients, many of whom
experience cutaneous irritation or inflammation. In fact, patients should be counseled
to limit facial washing and avoid harsh or abrasive cleansers, as these may exacerbate
acne. Cosmetics and emollients should be limited to those with noncomedogenic,
nonirritating, hypoallergenic properties. Daily moisturizers with sun protection factors
may alleviate dryness and prevent phototoxicity.
Explaining the risks, benefits, and possible side effects of acne medications empowers
patients to make informed decisions while enabling the clinician to assess the patient's
comfort level with various treatment options. Topical medications are often applied
inappropriately, as most patients erroneously treat only active lesions unless they
understand that topical medications also work by preventing new outbreaks. Thus,
topical acne medications should be applied to the entire affected area. The most
important cause of treatment failure in acne therapy is lack of compliance, which can
be minimized with careful instruction. Developing a close therapeutic relationship
with the acne patient is imperative.
Topical medications
Formulations: Topical acne medications are available in a variety of
formulations, including gels, creams, lotions, and solutions. Proper selection of
appropriate vehicles will minimize untoward side effects, thereby increasing
patient compliance and satisfaction. In general, patients with dry or sensitive
skin may benefit from the extra emollients found in a cream vehicle. Gels are
typically reserved for patients with oily skin, as they tend to be more drying and
may help reduce facial shine. Lotions are prescribed in a variety of settings.
Patients and the clinician alike often have preferences, and the ultimate decision
on topical formulations will depend largely on a combination of all these factors.
Topical acne medications are available in a variety of formulations, including gels,
creams, lotions, and solutions. Proper selection of appropriate vehicles will minimize
untoward side effects, thereby increasing patient compliance and satisfaction. In
general, patients with dry or sensitive skin may benefit from the extra emollients
found in a cream vehicle. Gels are typically reserved for patients with oily skin, as
they tend to be more drying and may help reduce facial shine. Lotions are prescribed
in a variety of settings. Patients and the clinician alike often have preferences, and the
ultimate decision on topical formulations will depend largely on a combination of all
these factors.

Retinoids: Retinoids and retinoid-analogs, including tretinoin, adapalene, and


tazarotene, prevent development of the microcomedo and are a reasonable first-line
medication for all acne variants. Most patients will remain on topical retinoids
throughout the treatment period. The primary action of retinoids is to reverse the
abnormal shedding of follicular keratinocytes, thereby preventing obstruction of the
pilosebaceous outlet. In addition, retinoids demonstrate well-documented antiinflammatory properties and may enhance absorption of other topical preparations
(e.g., antibiotics) by normalizing the follicular microenvironment.
Topical retinoids, which are available in multiple vehicles in a wide variety of
concentrations, can be drying and irritating; therefore, appropriate choice of
formulation may minimize the adverse effects. A derivative of vitamin A, tretinoin is
available as Retin-A in six strengths as a cream, gel, or liquid. Retin-A Micro (0.1%
or 0.04%), is a unique formulation in which the tretinoin is encapsulated in a porous
microsphere from which the medication is slowly released over time. This novel
delivery system and other methods reduce cutaneous irritation, potentially allowing
the patient to utilize higher concentrations with fewer adverse effects. One minor
limitation of the original tretinoin formulation is that it is unstable in sunlight; thus, it
is often applied at bedtime. The newer microencapsulated formulation of tretinoin is
more photostable.
Topical tretinoin and adapalene are classified as pregnancy category C by the FDA,
due to the lack of well-controlled clinical studies in pregnant women. Transdermal
absorption of topical tretinoin was studied, and the resultant systemic concentration
was below endogenous retinoid levels.14 Birth defects were reported in two patients
using topical tretinoin and one using adapalene, but causation was not proven.15
Tazarotene (Tazorac) is available as a cream or gel in either 0.05% or 0.1%
concentrations. Only the 0.1% concentration, a pregnancy category X drug, is FDAapproved for acne. Tazarotene may be more irritating than other retinoids; doserelated erythema and burning are the most common adverse effects. Like other topical
retinoids, tazarotene is efficacious in reducing both inflammatory and
noninflammatory lesions. Because tazarotene causes increased irritation, it is usually
considered a second-line retinoid option for patients who have failed tretinoin or
adapalene.
For all retinoids, visible improvement can occur as early as two weeks after beginning
treatment. There may be a slight worsening during the initial few weeks, as the
epidermis thins and allows previously unapparent lesions to rise to the skin surface.
Inactivated by UV light and oxidized by benzoyl peroxide, tretinoin should be applied
only at night and never with benzoyl peroxide. Topical retinoid decreases the amount
of native UV protection by thinning the stratum corneum; thus daily use of sunscreen
is recommended. The irritation associated with topical retinoids is dose-dependent and
will usually diminish with continued application. Reducing application frequency to
every other night during the first few weeks of treatment may minimize any initial
discomfort.
Topical antibiotics: Another mainstay of treatment, topical antibiotics are commonly
used in conjunction with retinoids for any degree of inflammatory acne. The most
commonly used are clindamycin and erythromycin, which are typically applied once
or twice daily. The most common side effects seen with topical antibiotics are mild
erythema or burning. Monotherapy with topical antibiotics may lead to bacterial
resistance and slower onset of therapeutic effect. Combinations of topical antibiotics
and benzoyl peroxide can increase efficacy and reduce antibiotic resistance to P.

acnes.16 Additionally, these combination products have been shown to be better


tolerated than benzoyl peroxide alone.17 Available in gel form, these products include
1% clindamycin/5% benzoyl peroxide (Benzaclin, Duac) and 3% erythromycin/5%
benzoyl peroxide (Benzamycin). Randomized controlled trials compared 1%
clindamycin/5% benzoyl peroxide with 1% clindamycin alone and found the
combination to be more efficacious with less P. acnes resistance.18

Acne: Choosing the best treatments


Other treatments: Azelaic acid (Azelex) is a dicarboxylic acid with bacteriostatic and
keratolytic properties. The 20% cream formulation is approved for acne; a 15% gel is
indicated only for rosacea. Azelaic acid may be particularly effective in the treatment
of cases with postinflammatory hyperpigmentation or concomitant melasma.
The utility of sodium sulfacetamide (Klaron) lotion, which is used once or twice daily,
may be attributed to its bacteriostatic properties. This agent is marketed for patients
with dry or sensitive skin, although studies to support the utility of sodium
sulfacetamide in this population are limited. Sodium sulfacetamide is contraindicated
in patients with sulfa allergies.
Nonprescription products may be used as primary or adjunctive treatments. Most acne
patients have used OTC products, such as benzoyl peroxide monotherapy (1%-10%)
and salicylic acid (2%). Used twice daily in a multitude of formulations, these
products are often a reasonable starting point for patients with mild acne. The utility
of benzoyl peroxide rests in its ability to inhibit P. acnes through oxidation, although
patients must realize that it will bleach fabrics. Salicylic acid is a weak keratolytic,
with a small potential to evoke contact dermatitis. It is commercially available in
numerous preparations, including washes, makeup, gels, lotions, and cleansers.
Systemic therapies
Oral antibiotics: In patients with mixed inflammatory and comedonal acne,
topical and oral medications used in combination are often necessary to achieve
maximal results. Topical retinoids in conjunction with topical or systemic
antibiotics have demonstrated greater utility than antimicrobials alone in both
comedonal and inflammatory lesions.19 Systemic antibiotics are considered firstline treatments for acne vulgaris. They decrease P. acnes colonization and also
have intrinsic anti-inflammatory effects.20 Commonly prescribed oral antibiotics
include erythromycin and tetracycline or its derivatives doxycycline and
minocycline.
In patients with mixed inflammatory and comedonal acne, topical and oral
medications used in combination are often necessary to achieve maximal results.
Topical retinoids in conjunction with topical or systemic antibiotics have
demonstrated greater utility than antimicrobials alone in both comedonal and
inflammatory lesions. Systemic antibiotics are considered first-line treatments for
acne vulgaris. They decrease colonization and also have intrinsic anti-inflammatory
effects. Commonly prescribed oral antibiotics include erythromycin and tetracycline
or its derivatives doxycycline and minocycline.
Erythromycin is a macrolide antibiotic that prevents the production of bacterial
proteins. Although the therapeutic dose is usually 500 mg twice daily, a dose of 250
mg twice daily may be used initially and adjusted as necessary. The safety and
tolerability profile of erythromycin are acceptable, with mild GI distress (the most
common side effect) generally mitigated by the co-administration of food; the FDA
classifies erythromycin as pregnancy category B. Unfortunately, P. acnes is quickly

becoming resistant to erythromycin, which frequently makes this medication a


second-line agent.

Tetracycline and its


derivatives also prevent protein synthesis. Its direct anti-inflammatory properties play
an important role in the therapeutic effect. The recommended starting dose is 500 mg
twice daily. Tetracycline has the inconvenience of having to be taken on an empty
stomach; patients should be instructed to take tetracycline one hour before or two
hours after meals. Polyvalent cations (calcium, zinc, and iron) decrease systemic
absorption. Tetracycline should not be used by pregnant women, or by patients
younger than 9 years old because of the risk of tooth discoloration and inhibited
skeletal growth. Moderate-to-severe phototoxicity (Figure 5) and GI intolerance may
also limit the use of the tetracyclines. Intracranial hypertension can result from any
medication in the tetracycline family, but this side effect may be more common if
tetracyclines are administered in conjunction with isotretinoin.21
Doxycycline, a lipophilic tetracycline derivative, is commonly prescribed for
moderate-to-severe acne vulgaris,
although photosensitivity may limit its usefulness. As with other tetracyclines, its
action is primarily bacteriostatic, but bacteriocidal activity might occur by
interference with bacterial protein synthesis. It is usually prescribed at a dose of 100
mg twice daily, although current studies are investigating the utility of smaller doses.
Minocycline is a potent acne medication but is typically reserved for patients who fail
or cannot tolerate the aforementioned oral antibiotics. Rare, but serious, side effects
are more common in patients taking minocycline than in those taking tetracycline or
doxycycline,22 including hyperpigmentation, drug-induced lupus, autoimmune
hepatitis, hypersensitivity reactions, and serum sicknesslike reactions. Vestibular
toxicity is more common in patients taking minocycline than other tetracyclines.
Minocycline is usually started at a dose of 100 mg twice daily.
Oral antibiotics must be taken for six to eight weeks before results are evident, and
treatment should continue for a minimum of six months to prevent resistance.23 Once
inflammation has resolved, oral antibiotics can be discontinued. During this
transitional period, topical antibiotics may be used to prevent flaring, and topical
retinoids should be continued. Some patients may require long-term oral antibiotic
therapy to control their acne and prevent hyperpigmentation and scarring.

There is controversy regarding the possible decreased efficacy of oral contraceptives


(OCs) in women taking systemic antibiotics. Although this concept is not supported
by research, some OC package inserts warn of decreased effectiveness if the
contraceptive is taken simultaneously with tetracyclines or ampicillin. Review of
pharmacokinetic data demonstrated a reduction of contraceptive steroid hormones
only with the concomitant use of rifampin. Nevertheless, it may be wise to inform
patients using oral antibiotics of this risk and recommend the use of a second method
of contraception during treatment with oral antibiotics. Women should also be warned
about the possible increased incidence of vaginal yeast infections during treatment.
Isotretinoin: Isotretinoin (Accutane, Claravis, Amnesteem, Sotret) is a systemic
vitamin A derivative used to treat severe nodulocystic and/or inflammatory acne.
Isotretinoin has utility against all four pathogenic factors contributing to acne, and its
mechanisms include induction of sebocyte atrophy, normalization of follicular
keratinocytes, and reduction of P. acnes colonization. It also has direct antiinflammatory effects. Isotretinoin is the only medication capable of providing a
sustained remission of acne. Usually prescribed at dosages of 40-80 mg daily, the rate
of absorption is increased with simultaneous ingestion of fatty foods. A total
cumulative dose of 120150 mg/kg has been shown to reduce the rate of relapse.24
The two most worrisome adverse effects of isotretinoin include severe birth defects
and depression. Severe fetal abnormalities involving several systems will result from
isotretinoin use during the gestational period. In an effort to educate patients
regarding these possibilities, the manufacturer of Accutane (which has been
withdrawn from the U.S. market) (Roche) developed the SMART program, which all
prescribers of Accutane were required to join. Manufacturers of generic isotretinoin
have similar programs. All these programs require two negative pregnancy tests prior
to prescribing isotretinoin in reproductive-aged women and mandate that all patients
use two forms of birth control throughout the treatment period and for one month
afterward. Urine pregnancy tests are checked at each monthly visit. Other possible
adverse effects of isotretinoin include hepatitis, hypertriglyceridemia, intracranial
hypertension, arthralgia, myalgias, night blindness, and hyperostoses. Serum liver
function tests and triglyceride levels are also monitored monthly during treatment.
The link between isotretinoin and depression is controversial. A meta-analysis
published in 2000 reviewed the purported risk of depression, suicide, or psychiatric
disorders in patients taking isotretinoin.25 In total, more than 7,500 patients on
isotretinoin were reviewed from Canadian and United Kingdom databases, and there
was no evidence that isotretinoin was associated with increased risk for depression,
suicide, or other psychiatric disorders. Several case reports and case series, however,
describe situations wherein depression occurred upon initiation and rechallenge with
isotretinoin.

Acne: Choosing the best treatments


Hormonal therapy: OCs may be a valuable adjunct in the treatment of acne in female
patients because OCs decrease circulating androgens, thereby reducing sebum
production. Estrogen decreases the secretion of gonadotropins by the anterior
pituitary, thereby reducing the amount of androgens produced by the ovaries. Finally,
estrogens in OCs increase the amount of sex hormone-binding globulin, which in turn
decreases free testosterone. Formulations that contain progestins with low androgenic
activity are the most efficacious for treating acne. Norgestimate/ethinyl estradiol
(Ortho Tri-Cyclen) and norethindrone acetate/ethinyl estradiol (Estrostep) are

currently FDA-approved for the treatment of moderate acne in females older than 15
years of age. Ortho Tri-Cyclen delivers a graduated dose of norgestimate with a
constant dose of estrogen. Estrostep provides a constant dose of norethindrone acetate
combined with a graduated dose of estrogen. Hormonal therapy is a good option for
many women, especially when oral contraception is desired. In the setting of
endocrine or hormonal irregularities, these medications can be extremely useful
interventions for the treatment of acne or other signs of androgen excess (e.g.,
menstrual irregularities).
In hyperandrogenic states, therapy is aimed at opposing the effects of androgen. This
is generally achieved by suppressing either adrenal or ovarian androgen production or
by blocking androgen receptors. Situations in which the excess androgen balance can
be corrected, such as hypersecretion by a tumor (removal of tumor), anorexia nervosa
(counseling and treatment of underlying disorder), and injection of anabolic
androgenic steroids (discontinuation of injections), should be treated appropriately.
Some progestins have intrinsic androgenic activity and therefore, progestin-only
contraceptives, such as medroxyprogesterone injections and levonorgestrel implants,
should be avoided. Besides OCs, other drugs that suppress androgen production or
block androgen receptors may be useful; however, due to their toxicity and limited
experience in controlled clinical trials, these drugs should be reserved for resistant
cases that are referred to a dermatologist, endocrinologist, or gynecologist with
experience using them.
Combination therapies
Because acne has a multifactorial pathogenesis, a drug combination that targets the
various steps in development seems rational. The combination of a topical retinoid
and an antibiotic (either topical or oral) is common. In fact, this combination provides
significantly greater and more rapid results than either agent used alone.26 Topical
retinoids in conjunction with benzoyl peroxide have also been used with success,
although one must keep in mind the incompatibility of benzoyl peroxide with tretinoin
and the need for separate application. To decrease the emergence of resistant strains of
P. acnes, topical antibiotic and benzoyl peroxide are useful. While topical retinoid,
antibiotic, and benzoyl peroxide are not often used simultaneously, sequential therapy
with tretinoin followed by an antibiotic-benzoyl peroxide combination has proved
successful.26
Surgical procedures
Comedo extraction for the treatment of comedonal acne is common and may be used
with topical retinoids. Extraction may be done manually or with light electrocautery.
Intralesional corticosteroid injections in minute amounts (25 mg/mL) are
instrumental for large and inflamed nodules. Chemical peels (trichloroacetic acid,
glycolic acids, salicylic acids) may also be effective against comedonal acne. Laser
resurfacing, dermabrasion, and subcision or punch grafting may reduce acne scarring.
Dermal augmentation with autologous or nonautologous tissue may enhance the
appearance of atrophic scars.
Photodynamic therapy is re-emerging as a useful option, particularly with use of blue
light (peak at 415 nm) or mixed red and blue light (peak 415 and 660 nm).27
Propionibacteria produce coproporphyrin III, which absorbs radiation at 415 nm.
Furthermore, 5-aminolevulinic acid is taken up by the pilosebaceous unit and
metabolized to protoporphyrin IX, which becomes phototoxic upon exposure to
visible light. Treatments are given once to twice weekly until clinical improvement,
which is usually seen within a few weeks.

The ClearLight System is a narrow-band (407-420 nm), high-intensity blue light


recently approved for use in moderate inflammatory acne. This treatment is given
twice weekly for approximately 10 minutes. Initial studies indicate a >80% response,
with a significant reduction of 59%-67% of inflammatory acne lesions after only eight
treatments of 8-15 minutes.27 Further studies are needed to assess duration of response
and cost-effectiveness of therapy.
Conclusion
Acne is a multifaceted skin disease that can have far-reaching consequences. The list
of severe and devastating side effects may be short, but it is impressive. Early
diagnosis and treatment are the mainstays of acne management and can prevent
scarring, postinflammatory hyperpigmentation, and physical and psychosocial
discomfort. The numerous drugs and technical modes available for the treatment of
acne are a testament to the varied needs of acne patients.
A balanced review of lesion type, patient requirements, and underlying expectations
can make the difference in controlling acne vulgaris.

Acute paronychia following cuticle


injury
Five months ago, a teenager suffered an injured cuticle while trimming his toenail.
Bacitracin and alcohol (recommended by a physician friend of the family) was
unsuccessful. The lesion oozes and bleeds after the patient wears shoes, and the
surrounding area is very tender and warm. What diagnosis and treatment should I
consider?
Maria Dellaplain, PA, Plano, Tex.
Inflammation of the nail fold (acute paronychia) can occur following injury to the
cuticle. Postinjury oozing and bleeding of the nail fold can also result from the
subsequent development of an ingrown toenail (onychocryptosis) or granulation tissue
(pyogenic granuloma). These conditions might require partial nail plate avulsion,
removal of the vascular tissue lesion, or both. Incision and drainage may be necessary
if abscess is present. Adjuvant topical care could include soaking in warm water and
mupirocin (Bactroban) 2% ointment. Systemic antibiotic treatment is likely to be
warranted as well. The initial choice of antimicrobial agent should cover
Staphylococcus aureus. However, bacterial culture may be helpful to definitively
identify the suspected organism(s) and guide subsequent management based on the
susceptibilities of the isolated bacteria.
Philip R. Cohen, MD
Bilateral pruritic papules on a woman's axillae
Diagnosis: Fox-Fordyce disease
Our patient had Fox-Fordyce disease, otherwise known as apocrine
miliaria. A chronic, recurrent, inflammatory disease affecting the sweat
glands, Fox-Fordyce disease is characterized by the presence of pruritic
follicular papules limited strictly to areas of the body with apocrine glands.
This uncommon disorder has a predilection for women, with a female-tomale ratio of 9:1. Onset usually occurs between the ages of 13 and 35
years. Only rarely has Fox-Fordyce been reported before puberty, in
postmenopausal women, or in men.

Typically Fox-Fordyce disease presents as skin-colored papules measuring


1-3 mm on a slightly erythematous base. The lesions occur in follicular (or
less commonly perifollicular) locations bearing apocrine glands. Besides
the axillae, Fox-Fordyce disease can be seen in the mammary areolae,
perineum, labia majora, and umbilicus. The lesions are pruritic; hairs tend
to be sparse in the affected areas, possibly due to the patient's rubbing
and scratching. The pruritus can be triggered by emotional,
pharmacologic, or physical factors that result in stimulation of apocrine
secretion. Absence of apocrine secretion from the affected glands is
common and attributed to obstruction of the apocrine duct at its entrance
into the follicular wall. When the disease affects a large proportion of the
apocrine glands, apocrine odor is reduced.
The exact etiology of Fox-Fordyce disease remains unclear, and no
connection with genetic factors, metabolic abnormalities, or endocrine
diseases has been established. Symptoms seem to improve in pregnant
women. This beneficial effect of pregnancy is particularly evident during
the last trimester.
Diagnosis of Fox-Fordyce disease is based on the presence of typical
clinical findings and confirmed on biopsy. The characteristic
histopathologic finding is vesicle formation in the intraepidermal portion of
the apocrine duct. The peripheral portion of the duct is plugged by keratin,
and dilation and rupture of the duct below this obstruction is seen. In time,
secondary changes, such as spongiosis and acanthosis of the epidermis in
the periductal area, along with inflammation of the dermis can be
detected.
Although the typical clinical features of this disease limit the differential
diagnosis, lichen simplex, lichen planus, and syringoma may be
considered.
The treatment of Fox-Fordyce disease is frequently unsatisfactory. Several
therapeutic approaches have been used, such as estrogens, topical and
systemic corticosteroids, steroid injections, symptomatic topical
treatment, topical tretinoin cream, and surgical excision. In our patient's
case, topical tretinoin relieved the pruritus.
A growing number of flesh-colored facial papules

Diagnosis: Trichoepitheliomas
Trichoepitheliomas are benign adnexal tumors with follicular germ-cell
differentiation. The classically skin-colored lesions can occur singly or in multiples.
They have a predilection for the central face, especially the nose, upper lip, and

cheeks. Our patient had multiple trichoepitheliomas, which developed during


.adolescence and then multiplied and coalesced into larger tumors
Once considered separate entities, trichoepitheliomas are now thought to be a variant
of trichoblastoma. Trichoblastomas are benign proliferations with follicular
germinative differentiation. Other variants of trichoblastoma include lymphadenoma,
trichogerminoma, and trichoblastic
.fibroma
Three forms of trichoepitheliomas
exist today: solitary, multiple, and
desmoplastic. Solitary
trichoepitheliomas are usually <1
cm skin-colored papules on the
face; however, lesions can
occasionally develop on other
anatomical sites, including the
.neck, trunk, and extremities
Multiple trichoepitheliomas begin
as discrete papules that tend to
proliferate around puberty and can
coalesce into larger tumorlike
nodules. Affected patients often
have an autosomal dominant trait
seen in Brooke-Spiegler syndrome (epithelioma adenoides cysticum). Patients with
this syndrome more commonly develop cylindromas, spiradenomas, and milia. In rare
instances, multiple trichoepitheliomas have been associated with systemic lupus
.erythematosus, myasthenia gravis, and Rombo syndrome
The third form, desmoplastic trichoepithelioma, is usually a firm papule or plaque
with a central depression found on the face, most commonly the cheek. These lesions
.can be erythematous rather than skin-colored
Under pathologic examination, a trichoepithelioma is a well-circumscribed lesion in
the dermis made up of predominantly follicular basaloid cells surrounded by a fibrous
stroma. Differentiating between trichoepitheliomas and basal cell carcinomas can
prove to be very difficult, and in rare instances, you may need to rely on
immunohistochemical studies.1
Trichoepitheliomas lack significant clefting between the basaloid cells and the stroma,
while in basal cell carcinomas, stromal clefting is a significant feature.1 The basaloid
cells are often arranged in a cerebriform pattern, but they can appear, for example, in
a reticulated pattern or as small clusters. Compared with other trichoepitheliomas,
.desmoplastic trichoepitheliomas have extensive stromal sclerosis
Although trichoepitheliomas are benign appendage tumors, the presence of both a
trichoepithelioma and a basal cell carcinoma in the same specimen has raised
questions regarding the potential for malignant transformation of trichoepitheliomas.

In one case report from Brazil, the malignant transformation of a giant solitary
trichoepithelioma into a basal cell carcinoma was identified using histopathologic
examination and the tissue expression of the bcl-2 oncogene.1 Treatment consisted of
.surgical excision and adjuvant radiation
Surgical excision is effective for solitary trichoepitheliomas. While the extra time and
cost of multiple excisions are deterrents, the main obstacle is patient discomfort and
scarring. Other treatment options include cryosurgery, electrosurgical destruction, and
laser destruction. Good cosmetic results have been reported with the erbium:yttriumaluminum-garnet and carbon dioxide lasers.2
Additionally, one case report describes the use of topical imiquimod (Aldara) in an
11-year-old girl with multiple trichoepitheliomas. The initial regimen of three
applications weekly was eventually increased to twice-daily use. Topical tretinoin gel
(Retin-A) was added to the therapy. The patient had 80% lesion clearance over
approximately three years.3 Therefore, while this therapy may not be a standard
approach, it may represent a good option for patients who wish to avoid scarring and
.are not bothered by treatment duration
A single nonpruritic growing erythematous plaque

Diagnosis: Erythema annulare centrifugum


The diagnosis was erythema annulare centrifugum (EAC), one of the figurative
erythemas characterized by annular, arciform, or polycyclic lesions. The classic case
begins with an annular erythematous plaque that migrates centrifugally with a trailing
scale. The erythema is due to dilation of the dermal blood vessels. There are no
systemic manifestations. The most common locations for EAC are the thighs and hips,
but any area of the glabrous skin can have lesions. The palms, soles, scalp, and
.mucous membranes are rarely involved
There are two major forms of EAC. The superficial variant (erythema gyratum
perstans) has the classic trailing white scale and is characterized by a mild superficial
perivascular lymphohistiocytic infiltrate that collects fairly tightly around the vessels.
The slightly raised advancing edge is caused by edema in the papillary dermis.
Lesions of the deep form have a more infiltrated border and tend to be elevated and
more indurated. A sharply demarcated perivascular infiltrate is seen in the mid and

lower dermis. EAC can appear at any age, although peak incidence is between the
.third and fifth decades. Incidence does not vary with sex
Both types of EAC lesions can follow a relapsing and chronic course of two years or
.longer, with individual lesions persisting for days or months
Alternatively, the eruption could complete its course in a few weeks. Autoimmune
bullous diseases, such as pemphigus, bullous pemphigoid, and linear immunoglobulin
A dermatosis, may initially manifest with EAC lesions that develop into typical bullae
.over the course of the disease
All EAC lesions begin as firm pink papules that expand centrifugally and develop
central clearing. Lesions can occur singly or in groups. The leading edge typically
advances 2-3 mm per day; within two weeks, a lesion can reach 6 cm. As the
expansion of the annular plaque is usually not uniform, incomplete arcs and
polycyclic lesions appear. Most lesions are at least minimally elevated, with
.desquamation at the inner margin
Lesions can be pruritic, and vesicles may be noted within the peripheral margin. In the
deep form of EAC, the advancing edges will be clearly elevated; there is typically no
.associated scale. Purpura can accompany the rash of either form
EAC is thought to be a vascular reaction pattern or hypersensivity reaction to various
antigens. The inflammation is thought to be a response to localized production of proinflammatory cytokines and vasoactive peptides. Post-interferon resolution of skin
lesions suggests a possible pathogenetic role for tumor necrosis factor- (TNF-a) and
.interleukin-2
EAC has been linked to infectious etiologies (dermatophytes, yeast, poxvirus,
Epstein-Barr virus, molluscum, and parasites) as well as medications, such as a
diuretic or antimalarial, cimet-idine, aldactone, salicylate, piroxicam, penicillin,
amitriptyline, or gold. Less common associations include particular foods,
autoimmune endocrinopathies, and neoplasms. Many reports have noted lesion
resolution with treatment of the underlying disease (if it has been identified). As a
.corollary, EAC flares have been correlated with recurrences of an underlying disorder
Conditions to be excluded prior to diagnosing EAC include urticaria, erythema
marginatum, erythema chronicum migrans, granuloma annulare, tinea corporis,
sarcoidosis, TB, pityriasis rosea, secondary syphilis, lichen planus, annular lupus
erythematosus, erythema multiforme, necrolytic migratory erythema, mycosis
fungoides, lymphoma cutis, seborrheic dermatitis, psoriasis, impetigo, Sneddon.Wilkinson disease, bullous pemphigoid, and elastosis perforans serpiginosa
Most lesions eventually resolve without scarring. Underlying disorders should be
corrected. While there is no specific therapy for EAC, topical steroids and/or
calcipotriol are routinely instituted, with occasional success. Vitamin D analogs
inhibit keratinocyte hyperproliferation, regulate epidermal differentiation, and inhibit
.cutaneous inflammation
Topical tacrolimus and pimecrolimus also have their supporters. Antihistamines
relieve pruritus. Many physicians also empirically try antibiotics and/or antifungal

agents. Systemic corticosteroids often yield a partial response of the dermatosis, but
.the eruption frequently recurs when treatment is stopped
Anecdotal reports suggest benefit with psoralen plus UVA and interferon-a. The
biologics, specifically etaner-cept, have also been reported to be successful. If EAC is
TH-1-mediated, with elevated levels of TNF- and associated pro-inflammatory
cytokines, anti-TNF- may be a treatment of choice if cost and side effects are not
.issues
Our patient's superficial EAC was attributed to divalproex. The lesions gradually
.subsided and disappeared two weeks after the medication was discontinued
A tender finger papule accompanied by fever and abdominal pain
BY TRAVIS W. VANDERGRIFF, MD, KIMBERLY M. NEYMAN, MD, AND BRIAN
DWINNELL, MD

Osler node associated with bacterial endocarditis


During history-taking, our patient reported three months of transient daily fevers. In
addition to the nodule on his finger, physical examination revealed a grade 3
.holosystolic murmur at the cardiac apex radiating to the axilla
Our patient had multisystemic signs and symptoms of bacterial endocarditis (BE).
Cutaneous manifestations of BE include Osler nodes, Janeway lesions, and splinter
.hemorrhages most commonly manifested in the nail bed
In 1909, Sir William Osler described the painful, erythematous nodules that occur on
the hands and feet of patients with BE.1 Up to 25% of BE patients will develop one or
more Osler nodes during the course of their illness. Classically, Osler nodes are
described as reddish-purple tender papules or nodules of the skin. They occur most
.often on the distal phalanges but may also be seen on the palms and soles
The lesions are characteristically painful; pain may precede the development of a
visible nodule by hours to days. Osler nodes tend to be self-limited and may last up to
several days before resolving. Histopathologically, Osler nodes are characterized by
leukocytoclastic and fibrinoid vasculitis. Cause of the nodes has
been postulated to be an immunologic response to bacterial
.antigens
The Janeway lesion, another cutaneous manifestation of BE, is
frequently confused with the Osler node. Janeway lesions are
classically described as small hemorrhagic red-to-blue macules
occurring on the palms and soles. They are characteristically
painless. This absence of pain is the most reliable factor distinguishing the Janeway
lesion from the Osler node.1
Additionally, Janeway lesions tend to be associated more closely with acute BE
(caused by virulent organisms such as Staphylococcus aureus), while Osler nodes
occur more often in the setting of subacute BE (caused by less virulent organisms).

Histopathologically, Janeway lesions are characterized by dermal microabscesses


.rather than vasculitis. Septic microemboli are thought to cause Janeway lesions
Gram-positive diplococci were identified in blood cultures from our patient.
Vegetations of the mitral valve were visualized on a transesophageal echocardiogram.
A CT scan of the abdomen revealed a peripheral wedge-shaped hypodensity
suggestive of an infarct due to septic embolization, accounting for the patient's acute
onset of abdominal pain. (See Medical Image of the Month.)
Our patient's gram-positive diplococci were ultimately determined to be viridans
group streptococci. He is currently being treated with a four-week course of IV
.nafcillin and is being evaluated for surgical valve replacement
Asymptomatic bilateral reddish lesions on the lower legs

Capillaritis
Our patient had Schamberg's disease, the most common type of capillaritis, also
known as pigmented purpuric dermatosis. Capillaritis is an extremely common
condition found primarily in adults. There are five subtypes of capillaritis:
Schamberg's disease, lichen aureus, Majocchi's disease (known as purpura annularis
telangiectodes and likely the rarest of all), pigmented purpuric dermatosis of Gougerot
and Blum, and pigmented purpuric dermatosis of Doucas and Kapetanakis. The exact
.etiology of capillaritis is unknown
Schamberg's capillaritis usually presents as asymptomatic pinpoint cayenne peppercolored macules within yellow-brown patches on the lower legs of older men,
although it can be seen in children as well. These purpuric macules may coalesce into
irregular small patches. The underlying yellow-brown patches represent older lesions
in which hemosiderin has replaced extravascular blood. The lesions tend to spread
proximally over time. Histologic findings seen in all types of capillaritis include
.extravascular RBCs, perivascular lymphocytes, and hemosiderin-laden macrophages
Lichen aureus presents, usually in children, as an asymptomatic golden-brown to
yellow patch that may contain petechiae. The Gougerot and Blum type may feature
lichenoid flat-topped papules in addition to purpuric macules. His- tologically, this
subtype includes a lichenoid dermatitis with basal-layer vacuolar alteration and a
bandlike lymphocytic infiltrate in addition to extravasation of RBCs. The Majocchi's
type clinically appears in adolescents and young women as annular purpuric macules
with telangiectasias in the borders of the lesion. The Doucas and Kapetanakis type

may have associated eczematous changes of the skin in addition to the characteristic
pinpoint purpura; the lesions are more papular than those of Schamberg's disease, and
there is some erythema and scaling. The eczematous changes seen clinically reflect
.the epidermal spongiosis (edema) seen histologically
The primary differential diagnosis for capillaritis is cutaneous small-vessel vasculitis,
also known as leukocytoclastic vasculitis. This eruption, commonly known as
palpable purpura, presents clinically as purpuric macules and papules on the lower
extremities in a symmetric distribution. Leukocytoclastic vasculitis has been
associated with a myriad of underlying diseasesfrom streptococcal infection to
systemic lupus erythematosus to malignancyso distinguishing this entity from
capillaritis is paramount. In contrast to the yellow-brown patches with cayennepepper red macules, leukocytoclastic vasculitis characteristically has a more purple
.color and larger palpable lesions, typically with an acute onset
The differential diagnosis of capillaritis may also include petechiae due to
thrombocytopenia of any cause, scurvy, and chronic stasis dermatitis. The clinical
history, distribution of the lesions, and absence of any other findings point toward the
.diagnosis of capillaritis
There is no universally effective therapy for capillaritis. Topical steroids may have
some benefit and are usually the first-line treatment for four to six weeks.
Pentoxifylline has also been tried. One case series considered oral rutoside. In our
patient, a course of topical triamcinolone ointment 0.1% twice daily for one month
.yielded little improvement, and therapy was discontinued
Asymptomatic cystic scrotal bumps of 40 years' duration

Idiopathic scrotal calcinosis


The patient had scrotal calcinosis, a rare condition characterized by multiple, slowly
growing, distinct nodular masses embedded within the dermis of the scrotal skin.

Uncertainty exists as to whether the condition is idiopathic or the result of dystrophic


.calcification of pre-existing epidermal cysts
Traditionally, calcium deposition within the body is thought to be a dystrophic or
metastatic process. Often seen at sites of previous inflammation or skin damage,
dystrophic calcification occurs in the setting of normal serum calcium and phosphate
levels and has been associated with connective tissue disease, such as scleroderma
.and polymyositis-dermatomyositis
Many doubt that calcinosis is idiopathic, citing evidence that calcification occurs
secondary to pre-existing epidermal cysts.1 Adequate data support the premise that
ruptured epithelial cysts often calcify. Other data show no evidence of pre-existing
cysts or of keratin epithelium near the calcified nodules.2
In contrast to dystrophic calcification of cysts, metastatic calcification has a
widespread distribution and is usually related to other conditions, such as
hyperparathyroidism, hypoparathyroidism, or renal disease. Calcium deposits occur
most often within visceral organs rather than in the skin or muscle. Another theory
links the pathogenesis of scrotal calcinosis to degenerative processes of the dartoic
muscle.3
The cause of the condition is most likely multifactorial. The proposition that the
calcinosis is caused by rupture of pre-existing epidermal cysts is valid but may apply
only to some cases. For the remainder, finding a pre-existing lesion is difficult, and
the term idiopathic may well be appropriate.4
There are no other diagnoses to be considered in a patient with multiple cysts. Some
men may have one or two cysts, but scrotal calcinosis is quite profound in having
many cysts. Our patient had none of the underlying diseases sometimes associated
with calcinosis. His multiple larger lesions were removed without complications using
.pinch-punch excisions
Asymptomatic erythematous plaque on the hand of a diabetic patient

Granuloma annulare
The diagnosis was granuloma annulare (GA), a common, benign skin condition with
asymptomatic skin-colored to violaceous papules that typically coalesce to form arciform or
annular plaques. Solitary umbilicated papules and nodules may also be seen, especially on
the fingers. Most GA patients are younger than 30 years; the female-to-male ratio is
approximately 2:1. While lesions of GA may occur anywhere on the body, they most
commonly arise at acral sites; 60% of cases are localized to the hands and arms and 20% to
.the legs and feet
A generalized form of the disease, which can occur in 15% of GA patients, is characterized by
innumerable small skin-colored papules or annular plaques on the trunk and extremities. This
type of GA has a later age of onset and poorer response to therapy. Other less common
variants are deep dermal or subcutaneous GA, which presents as large painless, skincolored, pseudorheumatoid nodules, and patch GA, which features erythematous patches on
.the trunk and extremities
The etiology of GA is unknown. Histologically, GA is a granulomatous dermatitis characterized
by interstitial lymphohistiocytic infiltrates, mucin deposition, and focal de-generation of
collagen and elastic fibers. In 25% of cases, well-formed palisaded granulomas consist of a
rim of histiocytes and lymphocytes surrounding a central zone of degenerated connective
.tissue and mucin
The condition has been theorized to be a T helper cell (Th1) inflammatory reaction, like TB,
i.e., lymphocytes that produce interferon-a orchestrate an inflammatory response that recruits
histiocytes to the dermis, where the subsequent release of lysosomal enzymes results in
damage to the surrounding connective tissue. The exact cause of this response is not known.
Most have no identifiable cause, although trauma, insect-bite reactions, tuberculin skin
.testing, and viral infections are some of the various inciting factors that have been implicated
An association between GA and diabetes mellitus has been proposed though not confirmed; a
number of reports and retrospective studies exist to support or refute the association. GA has
also been reported as a rare paraneoplastic, granulomatous reaction to solid-organ tumors,
.Hodgkin's disease, non-Hodgkin's lymphoma, and granulomatous mycosis fungoides
The duration of untreated lesions ranges from a few weeks to several decades, but on
average, spontaneous resolution is seen in 50% of cases within two years. The recurrence
rate of GA is approximately 40%; lesions often recur at the original site. Recurrent lesions

.tend to resolve more rapidly than primary lesions (80% within two years)
Given the self-limited and benign nature of the condition, treatment for a patient such as ours
with localized asymptomatic diseasegenerally consists of reassurance and observation. If
therapy is desired, high-potency topical steroids (with or without occlusion) or intralesional
steroid injections are considered first-line agents. Cryosurgery, psoralen-UVA phototherapy,
and carbon dioxide laser treatment have also been used. A variety of systemic agents,
including niacinamide, isotretinoin, antimalarials, cyclosporin A, chlorambucil, dapsone, and
pentoxyfylline, have been reported anecdotally as effective, but these are generally reserved
.for severe, refractory, and generalized cases

Back lesions with hairy clues

Case #1: Becker nevus

Occurring primarily in young men, Becker nevus (BN) generally appears first as an
irregular melanosis or hyperpigmentation on the shoulder, anterior chest, scapula, or
upper arm in neonates, infants, or children; only rarely does BN occur on the legs,
forearms, or face or in women. The benign lesion is usually unilateral and has
.geographic borders
Prevalence of BN has been reported as 0.2%-2.0%. Congenital and familial cases
have been noted. Bony and soft-tissue structural anomalies occur with increased
.frequency in BN patients
Hormones influence BN. In puberty and early adulthood, the lesion gradually enlarges
and becomes hairy. One investigator termed BN an androgen-mediated hyperplasia
with increased androgen receptors. Acneiform lesions and breast hypoplasia were
seen in one patient. Another report documented concomitant BN and acanthosis
.nigricans (which is linked to insulinlike growth factor)
Histologically, BN has three variants: (1) melanotic, (2) hypertrichotic, and (3) mixed.
The epidermis in a BN manifests with mild acanthosis and hyperkeratosis. In most

cases, rete ridges exhibit regular elongation. Melanin, but not the number of
melanocytes, is usually increased. The superficial dermis possesses melanophages.
Hypertrichosis in a BN reflects an increased number of morphologically normal
.follicular units. Most BN manifest a slight increase in dermal smooth muscles
Inflammatory diseases have been noted with BN. Concomitant lichen planus as well
as eczematous dermatitis, prurigo nodularis, granuloma annulare, and perforating
granulomatous folliculitis, each confined to a BN, have been noted. Noninflammatory dermatoses seen in association with BN include hypopigmented
pityriasis versicolor on a pre-existing BN and hypohidrosis and BN sharing the same
location. BN patients have also demonstrated chromosomal mosaicism and congenital
.adrenal hyperplasia
Breast defects have often been linked to BN. Ipsilateral breast hypoplasia (with and
without localized lipoatrophy) has been reported; spironolactone yielded some
improvement. Other defects include supernumerary nipples and ipsilateral areolar
hypoplasia along with limb asymmetry in three males with BN and one male with
.giant BN
BN can also be accompanied by other congenital hamartomas and abnormalities, such
as connective tissue nevi or smooth-muscle hamartomas. One patient had a port wine
stain (nevus flammeus), congenital BN, caf-au-lait-macule, and lentigines.
Lymphangioma with BN has been noted. Other defects seen with BN include
unilateral or ipsilateral pectoralis major aplasia, ipsilateral limb shortening, ipsilateral
foot enlargement, spina bifida, scoliosis, pectus carinatum, localized lipoatrophy
.polythelia, and accessory scrotum
Multiple or bilateral lesions are rare. BN may simulate congenital melanocytic nevus,
speckled lentiginous nevus, or nevus spilus. BN can be explained by paradominant
inheritance that is manifested clinically only by an acquired loss of heterozygosity. An
epidermal nevus on the left neck and a BN on the left shoulder led to speculation that
this supposed double mosaicism could exemplify twin spotting or non-allelic
.didymosis
Both erbium:yttrium-aluminium-garnet (Er:YAG) and neodymium:YAG (Nd:YAG)
lasers are safe for the treatment of BN. For pigment removal, one pass with an
Er:YAG is superior to three treatment sessions with the Nd:YAG laser. Fractional
resurfacing (the fraxel laser) and the ruby laser have also been used successfully to
treat BN. Wide surgical excision should be avoided as it can result in disfiguring
.scars
This patient decided to leave his BN alone when informed that laser treatment was not
.covered by insurance
Bilateral pruritic papules on a woman's axillae

Diagnosis: Fox-Fordyce disease


Our patient had Fox-Fordyce disease, otherwise known as apocrine
miliaria. A chronic, recurrent, inflammatory disease affecting the sweat
glands, Fox-Fordyce disease is characterized by the presence of pruritic
follicular papules limited strictly to areas of the body with apocrine glands.
This uncommon disorder has a predilection for women, with a female-tomale ratio of 9:1. Onset usually occurs between the ages of 13 and 35
years. Only rarely has Fox-Fordyce been reported before puberty, in
postmenopausal women, or in men.
Typically Fox-Fordyce disease presents as skin-colored papules measuring
1-3 mm on a slightly erythematous base. The lesions occur in follicular (or
less commonly perifollicular) locations bearing apocrine glands. Besides
the axillae, Fox-Fordyce disease can be seen in the mammary areolae,
perineum, labia majora, and umbilicus. The lesions are pruritic; hairs tend
to be sparse in the affected areas, possibly due to the patient's rubbing
and scratching. The pruritus can be triggered by emotional,
pharmacologic, or physical factors that result in stimulation of apocrine
secretion. Absence of apocrine secretion from the affected glands is
common and attributed to obstruction of the apocrine duct at its entrance
into the follicular wall. When the disease affects a large proportion of the
apocrine glands, apocrine odor is reduced.
The exact etiology of Fox-Fordyce disease remains unclear, and no
connection with genetic factors, metabolic abnormalities, or endocrine
diseases has been established. Symptoms seem to improve in pregnant
women. This beneficial effect of pregnancy is particularly evident during
the last trimester.
Diagnosis of Fox-Fordyce disease is based on the presence of typical
clinical findings and confirmed on biopsy. The characteristic
histopathologic finding is vesicle formation in the intraepidermal portion of
the apocrine duct. The peripheral portion of the duct is plugged by keratin,
and dilation and rupture of the duct below this obstruction is seen. In time,

secondary changes, such as spongiosis and acanthosis of the epidermis in


the periductal area, along with inflammation of the dermis can be
detected.
Although the typical clinical features of this disease limit the differential
diagnosis, lichen simplex, lichen planus, and syringoma may be
considered.
The treatment of Fox-Fordyce disease is frequently unsatisfactory. Several
therapeutic approaches have been used, such as estrogens, topical and
systemic corticosteroids, steroid injections, symptomatic topical
treatment, topical tretinoin cream, and surgical excision. In our patient's
case, topical tretinoin relieved the pruritus.
Brown bumps that evolved into verrucous plaques

Confluent and reticulated papillomatosis of Gougerot


and Carteaud
Confluent and reticulated papillomatosis (CARP), first described by Gougerot and Carteaud in 1927,
typically presents as small hyperpigmented papules on the central back or chest. These small papules
rapidly enlarge to become larger verrucous papules or coalesce into verrucous plaques. The plaques
then merge to form a reticular pattern. CARP commonly occurs during puberty. It has a preponderance
for women and blacks, being 2.5 times more common in women and twice as common in blacks.1
The pathogenesis of CARP is unknown. Among the theories proposed are endocrine imbalance, a
disorder of keratinization, and an atypical response to Pityrosporum orbiculare.1 However, none of
.these has been proven, and the majority of patients have no other abnormalities
The histopathologic picturehyperkeratosis, acanthosis, and papillomatosisis not specific for CARP
and, in the absence of any history, can be confused with that of epidermal nevus or acanthosis
nigricans. Clinically, however, the three are distinctly different: the epidermal nevus being a solitary
lesion and acanthosis nigricans being a more confluent hyperpigmentation. In some cases of CARP, a
.mild superficial perivascular lymphocytic infiltrate is seen
The treatment of CARP can be very disappointing, even for the seasoned dermatologist. A multitude of
treatments have proven unsuccessful. Even when CARP does respond, it often recurs after the
treatment is discontinued. Oral minocycline is frequently reported to be effective. One report of oral
minocycline monotherapy demonstrated resolution without recurrences in 50% of patients and disease-

free intervals as long as 18 months in others.2 Additional treatment options include topical salicylic
acid, hydroquinone, oral and topical antifungals, 5-fluorouracil, isotretinoin, acitretin, and various other
oral antibiotics.1
When counseling a patient who has CARP, it is very important to discuss the difficulty in management
and the possibility of recurrence after treatment is discontinued. Raising these issues up front will
.increase the likelihood of realistic expectations on the part of the patient
\A course of minocycline yielded a partial response that pleased my patien

?Can we expect a future decrease in patients with shingles


Q. Will the varicella vaccine lead to a significant reduction in the incidence
of shingles (herpes zoster) in future generations? Since the vaccine does
not actually cause the virus to go dormant in the spinal cord, can/will
these patients ever get shingles?
A. The varicella "chickenpox" vaccine (Varivax) is a live attenuated
vaccine that was licensed in 1995 and is effective in preventing primary
infection with wild-type varicella-zoster virus (VZV). Whether varicella
vaccination has an effect on the incidence of herpes zoster in individuals
who never received the Varivax varicella vaccine is unclear. An initial rise
in the incidence of herpes zoster was predicted secondary to the
anticipated fall in varicella incidence, and early studies supported this
hypothesis. However, a more recent investigation demonstrated that the
incidence of shingles has been stable as the incidence of chickenpox
declined between 1992 and 2002.
Herpes zoster represents clinical re-infection in persons who previously
had a primary varicella infection. Elevated cellular immune response
(reflected by increased VZV antibodies) confers protection from clinical reinfection (herpes zoster) in people who have previously had varicella
infection. Yet age younger than 1 year, mild varicella infection (without an
adequate immune response), and genetics are risk factors for the
development of herpes zostereven in immunocompetent individuals who
had a prior episode of chickenpox. However, herpes zoster vaccine
(Zostavax) is not recommended for people of any age who previously
received varicella vaccine. Hence continued surveillance is warranted to
observe whether immunocompetent individuals who receive Varivax
subsequently develop shingles
Clinical Quiz: Dermatology Dx

Figure 1

Figure 2

Dermatomyositis
Our patient had dermatomyositis, a connective tissue/vascular disorder of uncertain etiology
that primarily affects the skin and skeletal muscles but can also involve the joints, esophagus,
heart, and lungs. The most common presenting complaint is symmetrical proximal muscle
weakness of the extremities, as evidenced by difficulty in rising from a chair, climbing steps,
or raising one's arms above the head. Fifty percent of patients experience the cutaneous
features concomitantly with the development of muscle complaints. In the remaining cases,
.the skin rashes precede the onset of muscle symptoms by weeks to as long as six years

Dermatomyositis can surface at any age; the highest incidence occurs in the fifth and sixth
decades. The condition is thought to result from an autoimmune process in which the
autoantigen targets the endothelium of endomysial capillaries. This leads to vascular
.compromise in the muscles and weakness
The incidence of cancer is increased with dermatomyositis in adults. The most common forms
of malignancy are ovarian, GI, pulmonary, and breast as well as non-Hodgkin's lymphoma.
Advanced age, skin involvement, and limited improvement with oral steroid therapy are the
.predominant risk factors for malignancy
Of the five classic cutaneous features of dermatomyositis, two are pathognomonic. Gottron's
papules are violaceous flat-topped papules on the dorsolateral interphalangeal and/or
metacarpophalangeal joints. Central atrophy, telangiectasis, and hypopigmentation are often
associated with these papules. Gottron's sign is a symmetric macular violaceous erythema,
with or without edema, overlying the dorsal aspect of the interphalangeal joints of the hands,
olecranon processes, patellas, and medial malleoli. The other three characteristic signs
include (1) a violaceous symmetrical erythema involving the dorsal aspect of the hands,
forearms, shoulders, forehead, V area of the neck, and upper chest, (2) the heliotrope rash, a
periorbital violaceous erythema accompanied by edema, and (3) periungual telangiectasia
with accompanying dystrophy of the cuticles. Less often, patients will have poikiloderma (with
skin epidermal atrophy, hyperpigmentation, hypopigmentation, and telangiectasia),
subepidermal bullous lesions and superficial erosions, palmar erythema, and calcinosis cutis
.or calcium deposition in muscles and tendons
Severity of skin findings and extent of muscle disease need not correlate, but muscle

pathology is essential to the diagnosis. Indeed, four of the five diagnostic criteria consist of
muscle signs and symptoms, including clinical progressive proximal muscle weakness;
histopathologic muscle findings; abnormal electromyogram; and elevated muscle enzyme
.levels
Tests helpful for diagnosis and evaluation of disease severity are electromyography, muscle
biopsy, serum muscle enzymes, MRI, chest radiograph, pulmonary function studies, barium
swallow, esophageal motility studies, electrocardiography, and serology. Blood tests include
serum aldolase, creatine kinase (CK), aminotransferases, and lactic dehydrogenase.
Antinuclear antibody (ANA) testing is positive in only 15% of patients. Precipitating
autoantibodies, referred to as myositis-associated antibodies and specific for
dermatomyositis, are detectable in 25% of patients. In adults, the malignancy evaluation may
include CT and age-appropriate cancer screening of the breast, pelvis, and rectum as well as
.colonoscopy
The differential diagnosis for dermatomyositis encompasses sarcoidosis, lupus, active muscle
infection, peripheral or central nervous system involvement, drug- or toxin-related muscle
.injury, metabolic disorders, endocrinopathy, myasthenia gravis, and muscular dystrophy
Treatment usually begins with high-dose oral prednisone. Muscle-enzyme levels will assess
therapeutic response. For example, CK ideally falls to 50% of pretreatment values within the
first month of therapy. CK levels are usually normal within three to four months. Clinically,
there is often improvement in muscle strength after two months of therapy. Most patients
require chronic use of prednisone to keep their condition under control. Those who do not
respond to steroids after three months require additional therapy with methotrexate,
.azathioprine, mycophenolate mofetil, cyclophosphamide, chlorambucil, or IV immunoglobulin
Our patient had elevated muscle enzymes and a slightly positive ANA with a speckled pattern.
Oral prednisone was prescribed for her muscle involvement and proved curative for her
ocular symptoms, which required only petroleum jelly. She chose not to treat her Gottron's
.papules

Diagnosing two distinct red rashes


CASE #1: Leukocytoclastic vasculitis

Leukocytoclastic vasculitis is characterized by inflammation directed at small blood


vessels in the skin. The process is mediated by neutrophils. Histologically,
leukocytoclasis (i.e., destruction of neutrophils leaving nuclear debris) can be seen.1,2,3
When blood vessel inflammation occurs, vessel wall destruction can take place, which
may lead to hemorrhage, ischemia, and/or infarction.2
Leukocytoclastic vasculitis may be the result of an idiopathic or primary process.
Henoch-Schonlein purpura is an idiopathic syndrome associated with a cutaneous
vasculitis as well as arthralgias and vasculitis in the renal and GI systems.2
Leukocytoclastic vasculitis may also result from a secondary process associated with
another systemic disease, such as infection, collagen vascular disease (particularly
systemic lupus erythematosus and rheumatoid arthritis), cryoglobulinemia, or,
occasionally, lymphoma and myeloma. It is important to be vigilant about noticing the
signs and symptoms of systemic disease in a patient presenting with a cutaneous

vasculitis. Although it is uncommon, recent drug ingestion is another possible


etiology of leukocytoclastic vasculitis. The drugs most frequently associated include
antibiotics (i.e., penicillin, sulfonamides, chloramphenicol, and streptomycin), aspirin,
.phenothiazine, and thiazides

Clinically,
leukocytoclastic vasculitis most often presents with palpable purpura, especially on
the lower extremities.2 Other less common presentations include nonpalpable purpura,
infiltrated erythema, ulcers, livedo reticularis, and nodules. Vasculitis that is restricted
to the skin is often referred to as cutaneous leukocytoclastic angiitis or
hypersensitivity vasculitis. Symptoms of a systemic involvement may include
hematuria, arthritis, fever, abdominal pain, melena, cough, hemoptysis, headaches,
and peripheral neuropathy. The kidney is the most frequently involved internal organ,
with some studies showing up to half of patients presenting with leukocytoclastic
vasculitis having renal involvement. Although chronic cutaneous disease may involve
ulceration or painful bouts of purpura, serious complications of leukocytoclastic
vasculitis are most often associated with internal organ involvement. When a patient
.with vasculitis presents, it is imperative to rule out systemic disease
Skin biopsy is the gold standard for diagnosing this condition. The biopsy should be
taken from the earliest purpuric or symptomatic lesion to obtain the most accurate
findings. If the biopsy is poorly timed, the pathological features of vasculitis may be
absent. This has to be considered in the interpretation of a negative biopsy from a
patient whose clinical findings suggest a cutaneous vasculitis. The diagnosis of
leukocytoclastic vasculitis can be confidently made when the inflammation directed at
small blood vessels in the skin is accompanied by fibrinoid necrosis. Endothelial cell
swelling is a sign of endothelial damage. Vascular damage can be inferred with the
extravasation of RBCs (purpura) and necrosis (infarct); however, these two findings
.are supportive rather than diagnostic (they are also seen in other disorders)
Direct immunofluorescence stains are helpful in diagnosing Henoch-Schonlein
purpura, in which the immunoglobulin (Ig) deposition is IgA. Henoch-Schonlein
purpura is essentially a leukocytoclastic vasculitis accompanied by IgA immune

complexes within affected organs. In adults, confirmation of the diagnosis by biopsy


and direct immunofluorescence is important, as Henoch-Schonlein purpura is not
common in this patient pouplation. However, in children, biopsy is reserved for
patients with an unusual presentation or those with significant renal disease
To rule out sepsis, blood cultures should be obtained in all patients presenting with a
cutaneous vasculitis and fever. If sepsis is suspected as an underlying etiology for
leukocytoclastic vasculitis, it is important to keep disseminated intravascular
coagulation (DIC) in the differential diagnosis. In DIC, purpuric lesions may be
elevated because of edema that acutely accompanies necrosis. Typically, patients with
DIC will have moderate to severe thrombocytopenia. Diagnosis can be confirmed by
demonstrating decreased fibrinogen and increased fibrinolysis (e.g., elevated fibrin
.degradation products and elevated D-dimer)
If no obvious etiology is apparent, patients with a cutaneous vasculitis should undergo
such serologic studies as antinuclear antibody, antineutrophil cytoplasmic antibody,
rheumatoid factor, serum protein electrophoresis, cryoglobulins, and hepatitis.associated antigen
If an underlying systemic disorder can be identified, the treatment of this underlying
disease may result in clearing of the cutaneous vasculitis. Similarly, if drug ingestion
or infection is the suspected etiology, withdrawal of the drug or treatment of the
infection may simultaneously treat the leukocytoclastic vasculitis. In patients without
systemic involvement, conservative treatment usually leads to good results.
Leukocytoclastic vasculitis often affects dependent areas, so elevating the legs or
wearing support hose may be useful. Therapy with antihistamines, aspirin, or
nonsteroidal anti-inflammatory drugs can be used. More extensive therapy is
indicated for recurrent or persistent skin disease. For mild chronic disease, colchicine
and dapsone are first-line agents. Severe cutaneous disease requires more potent
immunosuppression. If the leukocytoclastic vasculitis is refractory, plasmapheresis
.and IV Ig are possible considerations
Leukocytoclastic vasculitis is an uncommon disorder. The annual incidence of biopsyproven cutaneous vasculitis ranges in studies from 39.6 to 59.8 per million. Although
leukocytoclastic vasculitis may affect anyone, it is slightly more prevalent in females.
The condition is also more prevalent in adults. When children are affected, it is most
.commonly Henoch-Schonlein purpura
Diagnosing two distinct red rashes
CASE #2: Benign pigmented purpura

Also known as pigmented purpuric dermatoses, benign pigmented purpura (BPP)


include a spectrum of disorders characterized clinically by a purpura that
morphologically appears to have distinct forms but histologically appears
indistinguishable. BPP can be seen in the following conditions: Schamberg's disease,
Majocchi's disease,4 lichen aureus, pigmented purpuric lichenoid dermatosis of
Gougerot and Blum, eczematid-like purpura of Doucas and Kapetanakis, and itching
purpura of Loewenthal. These dermatoses can manifest with petechiae and/or purpura

and are usually found on the lower legs, but any area of the body can be affected. BPP
is not associated with serious sequelae. However, the condition can persist for years
and may be associated with such troubling symptoms as itching and patient distress
over the cosmetic appearance.5

The lesions in
Schamberg's disease have been likened in appearance to cayenne pepper because of
the pinhead-sized reddish puncta. Another name for Schamberg's disease is
progressive pigmented dermatosis, as these puncta can form irregular plaques of
orange or brown pigmentation. Majocchi's disease is most common in young women.
Lesions in Majocchi disease are punctate telangiectatic macules progressing to
annular, hyperpigmented patches; thus, Majocchi's disease is also known as purpura
annularis telangiectodes. The term lichen aureus describes the lichenoid macules and
papules and golden-bronze color of the typical eruption. Lesions in pigmented
purpuric lichenoid dermatosis of Gougerot and Blum are minute, lichenoid papules
that tend to fuse into plaques of various hues. Eczematid-like purpura of Doucas and
Kapetanakis is characterized by purpura with scale on the surface. In itching purpura
of Loewenthal, the lesions are more extensive, and patients typically complain of
.severe pruritus
The etiologies of these conditions of the pigmented purpuric dermatoses are unknown,
but a common pathogenetic mechanism is suspected. These conditions have a similar
histological presentationa perivascular infiltrate composed of T-cell lymphocytes
and macrophages centered on the superficial small blood vessels of the skin. Signs of
endothelial cell swelling and narrowing of the lumen are typically seen. Extravasation
of red blood cells (resulting in purpura) and marked hemosiderin deposition in
.macrophages are also found
Dermoscopy has been reported to be a useful tool for assisting in the clinical
diagnosis of BPP.6 A complete blood count and peripheral smear should be done to
rule out thrombocytopenia. Prothrombin time and partial thromboplastin time may
also be done to exclude other possible causes of purpura. Tests for antinuclear

antibody; rheumatoid factor; human type B hepatitis, surface antigen antibody; and
.anti-hepatitis C virus antibody should also be considered
Extensive pigmented purpura should be biopsied. Skin biopsy helps confirm the
diagnosis of BPP and exclude mycosis fungoides, which in its early stages may
closely resemble a pigmented purpuric dermatitis. Despite its name, mycosis
fungoides is not a fungal infection but rather a cutaneous T-cell lymphoma. The skin
lesions in mycosis fungoides result from a proliferation in the dermis of malignant T
lymphocytes, which have a tendency to migrate into the epidermis. Skin biopsy of the
lesions demonstrate small-to medium-sized atypical lymphocytes with highly
convoluted or cerebriform nuclei infiltrating the upper dermis. Skin biopsy may also
demonstrate Pautrier microabscesses (collections of atypical lymphocytes), which are
pathognomonic for mycosis fungoides. The prognosis of mycosis fungoides varies
with disease stage. In most patients, mycosis fungoides is a chronic disease with a
.slow progression over many years
BPP may resolve spontaneously, and no medical intervention has been proven to treat
BPP. Compression stockings may help by decreasing venous stasis and edema.
Topical steroids are recommended to help control BPP and may also offer relief from
pruritus. Psoralen photochemotherapy has had beneficial results in several case
reports. Such systemic therapies as methotrexate should be reserved for patients with
.highly symptomatic disease refractory to other treatments
The patient in this case was treated with triamcinolone ointment 0.1% with partial
improvement over two months. N
Differentiating cutaneous dells

CASE #1
Punctate keratoderma of the palm and sole
Palmoplantar keratodermas (PPKs) are divided into three types. Type I (the
form most likely to be affecting our patient) is punctate keratoderma,
which manifests with numerous hard wartlike dells on the palms or soles.
The lesions range from 0.1 to 2 mm in diameter and depth. Onset is
between the ages of 12 and 20 years; incidence is 1-1.7/100,000 persons.
Type II PPK, or filiform keratoderma, features spicules that resemble music
box spines on the palms and soles, although the disorder can also occur
elsewhere on the body. Onset usually occurs between the ages of 12 and
50 years. Type III PPK, or marginal keratoderma, manifests with oval or

polygonal keratotic dells on the borders of the hands, feet, and wrists as
well as in the center of the palms and soles. The disorder usually appears
in late childhood or early adulthood. Type III PPK can be associated with
diffuse palmoplantar keratoderma, hypohidrosis, and nail abnormalities.
Focal hyperkeratosis can be seen at the pressure points, due to
mechanical rubbing, or in areas of the skin that are irritated.
Type I PPK can be acquired or hereditary. Acquired punctate keratoderma
has a variety of causes ranging from essential (idiopathic/noninherited) to
neoplastic. Punctate PPK of the palmar creases (PPPK[PC]) occurs in black
patients and is likely the most common type of essential PPPK. (The
disorder may have a genetic basis, but that has not been defined.) Other
names for PPPK(PC) include keratotic pits of the palmar creases, punctate
keratosis of the palmar creases, hyperkeratosis punctata of the palmar
creases, keratosis punctata, keratoderma punctata, hyperkeratosis
penetrans, and lenticular atrophia of the palmar crease. PPPK(PC) has
been said by some to have associations with atopy, knuckle pads,
dermatitis herpetiformis, striate keratoderma, psoriasis, Dupuytren
contractures, pterygium inversum unguis, and focal acral hyperkeratosis.
Acquired punctate keratodermas caused by arsenic are termed arsenical
keratoses. Arsenical keratoses look like multiple hypertrophic actinic
keratoses on the palms and soles (an uncommon location for actinic
keratoses). Arsenic is a systemic carcinogen, and arsenical keratoses can
manifest contemporaneously with angiosarcoma of the liver, squamous
cell cancer of the skin, and bronchial adenocarcinoma. Old age and
internal malignancies can correlate with idiopathic palmoplantar
keratoderma. Breast, renal, colon, and lung
cancer can be associated with idiopathic porokeratotic palmoplantar
keratoderma, a condition that has been liked to secondary syphilis.
Inherited punctate keratodermas are one of the three types of inherited
palmoplantar keratoderma that include focal, diffuse, and punctate
varieties. There are three categories of hereditary punctate PPKs: type I
includes Buschke-Fischer-Brauer disease, keratosis punctata, and keratosis
papulosa; type II is porokeratosis punctata palmaris et plantaris, and type
III comprises acrokeratoelastoidosis lichenoides. Punctate PPK affects men
and women equally in an autosomal dominant pattern with variable
penetrance. No specific genes have been linked to inherited PPKs,
although some linkages (e.g., 18q24.13-8q24.21 and 15q22-15q24) for
types I have been suggested.
Wood's light can excite white fluorescence of type I hereditary PPK.
Compared with normal controls, kindred with type I PPK seem to suffer
more malignancies (adenocarcinomas of the kidney, breast, pancreas, and
colon and Hodgkin disease). Histopathologic examination can demonstrate
hyperkeratotic epidermis without columns of parakeratosis or

elastorrhexis. On electron microscopy, the basal cells of the epidermis may


contain enlarged nucleoli and abundant tonofilaments, with
keratohyalinlike granules confined to the upper part of the stratum
spinosum.
First-line treatment of all types of PPK usually involves moisturizers that
contain natural moisturizing agents, such as lactic acid (12%) and urea
(compositions that are 40% or higher). Salicyclic acid 3% and 6% can be
helpful as well. Also beneficial are topical retinoids (adapalene [Differin],
tretinoin, and tazarotene); in severe cases, acitretin can be used. Some
dermatologists suggest that only mechanical debridement and excision
are effective for PPK.
Differentiating cutaneous dells

CASE #2
Myrmecia

Our patient had deep-seated warts, also known as myrmecia. The warts, which are
caused by the type 1 human papillomavirus (HPV), manifest as delled (centrally
depressed) hyperkeratotic lesions thought by some to resemble an anthill. (Bulldog
ants belonging to the species Myrmecia and endemic to Australia inhabit delled
mounds, lending their name to the disorder.) On histology, myrmecia can have an
endophytic epidermal growth pattern and possess virion-loaded basophilic nuclear
inclusions and basophilic parakeratotic cells in the upper epidermis. Abundant
polygonal, refractile-appearing eosinophilic inclusions (ring formations of keratin
filaments) associated with HPV 1 E4 gene products (a 17-kDa E1-E4 fusion protein
and a 16-kDa species lacking the five E1 amino acids and a few E4 residues) are also
.apparent
The body areas most commonly affected by warts are the hands and feet. Some
studies suggest that HPV types 2, 27, and 57 cause the majority of palmoplantar
warts. HPV types 1, 2, 3, 4, 27, 29, and 57 have been noted to be a significant cause

.of warts elsewhere on the body


One of the most common skin infections, warts are estimated to occur in at least 10%
of the population and account for approximately 10% of visits to physicians by
patients seeking dermatologic care. The morbidity associated with warts is not
insignificant because of the pain they induce when they occur on the feet and the
emotional distress they engender when they occur on the hands, as they are visible to
.others during hand shaking
Besides myrmecia, other types of warts occur on the feet. HPV 4, 60, 63, and 65
induce plantar cysts that form inclusion warts of the sole. Any type of plantar wart can
.coalesce into large plaques termed mosaic warts
When a patient presents with a verrucous lesion, the most common misdiagnosis for a
wart is a callus. Three clinical findings distinguish warts from calluses: (1) Warts
commonly possess visible black dots, which represent thrombosed capillaries. Skin
cells affected by warts secrete a factor that induces the formation of additional but not
necessarily functional blood vessels. Calluses do not exhibit black dots. (2) Warts
often break the continuity of the skin lines (dermatoglyphics), while calluses do not.
(3) Warts are frequently painful when subjected to direct and lateral pressure, but
.calluses are often painful only to direct pressure
The ideal treatment for palmoplantar warts has yet to be devised. Warts can disappear
spontaneously. The larger their size and the longer they have been present, the less
likely warts are to disappear without treatment. Cyrotherapy and keratolytic topical
agents (salicylic acid in the form of poultices or a liquid) are the mainstay of
treatment. A 10-second exposure to liquid nitrogen has been found to be more
effective, albeit with a higher incidence of side effects, than shorter durations of liquid
.nitrogen exposure
Some interesting differences in treatment efficacy between plantar and hand warts
have emerged, based on clinical trials and other therapeutic approaches. One open,
randomized, parallel-group study found that two freeze-thaw cycles were more
effective than cryotherapy at three-week intervals for plantar but not hand warts.1
Paring improves the cure rate for plantar warts but not for hand warts.2 Bleomycin in
solution is a useful treatment for recalcitrant warts (I think particularly for plantar
warts), but it has a variety of side effects, including possible pain during and after
treatment, pigment changes, purple digit, Raynaud phenomenon, necrotic eschar,
scarring, and nail damage.3
Discrete yellow lesions on the eyelid

Xanthelasma
Our patient had xanthelasma, or xanthelasma palpebrarum, oval or elongated
yellowish plaques just beneath the periorbital skin. The plaques occur most commonly
near the inner canthus of the eyelid, more often on the upper lid than the lower lid.
Frequently, the lesions are symmetrical; often all four lids are involved. On palpation,
xanthelasma may have a soft, semisolid, or calcified texture. The lesions are neither
inflammatory nor painful, and there is no tendency toward malignancy. Xanthelasma
.plaques have a tendency to progress, coalesce, and become permanent
Histologically, xanthelasma lesions reveal accumulations of lipid-laden macrophages,
termed histiocytes and/or xanthoma cells. These foamy histiocytes reside
primarily within the upper reticular dermis. The main lipid stored in both
hyperlipidemic and normolipidemic xanthelasmas is cholesterol, most of which is
.esterified
Our patient related an episode of allergic dermatitis with the onset of her xanthelasma.
Although no precipitating factor is normally associated with this condition,
xanthelasma has been reported following erythroderma and inflammatory skin
disorders. For example, two prominent British dermatologists reported the condition
in a patient following severe facial allergic contact dermatitis from pphenylenediamine in a black eyelash-tinting product.1 The mechanism by which
macrophage accumulation, cholesterol uptake, and foam-cell formation would occur
following an inflammatory condition is not understood; however, it has been
suggested that increased plasma lipid peroxidation, derived from oxidized low-density
lipoprotein, may lead to accumulation of cholesterol in macrophages and formation of
foam cells.1
Xanthelasma lesions are the most common type of cutaneous xanthoma. Prevalence,
which increases with age, is estimated to be 1.1% in women and 0.3% in men. In most
.cases, no other xanthomas of any type are found anywhere else on the body
Approximately one half of patients with xanthelasma have elevated plasma lipid
levels. Some affected individuals may have type II hyperlipidemia. Others may have
primary genetic disease, such as familial dyslipoproteinemia, familial

hypertriglyceridemia, and familial lipoprotein lipase deficiency. Two common causes


of secondary hyperlipidemia are diabetes and cirrhosis. As a rule of thumb, patients
with xanthelasma in the absence of a family history of the disorder are more likely to
.have elevated serum cholesterol and atherosclerotic disease
The other 50% of xanthelasma patients are normolipidemic, although they may
demonstrate some minor alteration in lipoproteins, such as abnormal distribution of
apolipoprotein (apo) E phenotypes, excessive production of apo B, decreased levels of
.HDL cholesterol, and/or a decrease in the ratio of HDL-to-LDL cholesterol
Patients with xanthelasma are typically in their fourth or fifth decade of life, but age at
onset can range from 15 to 75 years. Of note, younger individuals with xanthelasma
have a proportionally greater likelihood of hyperlipidemia and hypercholesterolemia
.than their older counterparts
Most patients with xanthelasma are asymptomatic, and their complaints generally
concern esthetics. Rarely, abnormally large xanthelasmas can affect eyelid function,
.causing ptosis or lagophthlamos
Xanthelasma is usually an obvious clinical diagnosis, but other lesions can simulate
its appearance and may be associated with disorders of a more serious nature. If there
is any doubt, surgical excision and pathologic analysis should be performed. A
differential diagnosis of xanthelasma could include: necrobiotic xanthogranuloma,
tuberous xanthomata, diffuse planar xanthoma, orbital lipogranulomata, juvenile
xanthogranulomata, Wegener granulomatosis, lipoid proteinosis, primary systemic
.amyloidosis, necrobiosis lipoidica, sarcoid, and atypical lymphoid infiltrate
In terms of medical workup, a serum lipid profile is recommended, including LDL
and HDL cholesterol levels. Because other metabolic disorders, particularly diabetes
and cirrhosis of the liver, can lead to increased serum lipids, determinations of fasting
.plasma glucose and liver function tests should also be considered
Recently, ultrasonographic measurement of intima-media thickness has revealed
premature carotid atherosclerosis in patients with normolipidemic and hyperlipidemic
xanthelasma.2 These findings suggest that patients with xanthelasma should be
considered to have an increased risk of cardiovascular disease independent of their
plasma lipid levels.2
Another report did not concur that normolipidemia with xanthelasma had a significant
association with carotid atherosclerosis, but this group of patients did demonstrate
some adverse cardiovascular profiles, namely higher BMI, waist circumference, and
LDL cholesterol.3 Dietary restriction and pharmacologic reduction of serum lipids,
although important in the overall care of a patient with abnormal lipids, show only
.limited response in cases of xanthelasma

Treatment of xanthelasma is primarily for cosmetic reasons. Numerous options are


available for the removal of xanthelasma palpebrarum, including surgical excision,
argon or carbon dioxide laser ablation, chemical cauterization (i.e., trichloroacetic
acid peel), electrodesiccation, and cryotherapy. Surgical treatments can potentially
cause scarring, irritation of the conjunctiva, and cutaneous pigment changes. No
single technique has emerged as the most effective. Moreover, recurrence rates of
about 40% are common regardless of removal method. Of these failures, 25% of cases
recurred within the first year; recurrences were more likely in patients with
.hyperlipidemia syndromes and in those whose xanthelasma involved all four eyelids
Our patient's blood lipids revealed cholesterol and triglyceride levels within normal
limits. Because of the present confusion as to whether normolipidemic patients with
xanthelasma have an increased risk of carotid atherosclerosis, I wrote to her primarycare clinician regarding the patient's possible need for cardiac workup. In terms of
treatment, the patient did not want chemical removal, but she was interested in
obtaining more information on laser therapy. I provided her with the names of two
clinicians who had a Q-switched neodymium:yttrium-aluminum-garnet laser and an
.argon laser, respectively
Disfiguring violaceous papules on a woman's nose

Lupus pernio
A punch biopsy of one papule revealed the classic noncaseating, naked granulomas of
sarcoidosis, and a chest x-ray demonstrated bilateral hilar lymphadenopathy. The skin
lesions were consistent with lupus pernio, one of the characteristic cutaneous
.manifestations of sarcoidosis
Lupus pernio is characterized by purple-red papules, nodules, or plaques usually seen
on the nose but sometimes found on the earlobes, fingers, and toes. Recognition of
this skin manifestation is important because 75% of lupus pernio patients have
.sarcoidosis of the lung
Although most cutaneous lesions of sarcoid heal without scarring, lupus pernio is an
exception and can be disfiguring. When the tip of the nose is involved, the nasal bones
as well as the mucosa can be affected, producing bony lytic lesions and obstructive

.symptoms
The term lupus pernio is confusing because the condition is not related to lupus
erythematosus. In addition, the term pernio refers to a condition characterized by
inflammatory papules that occur on the fingertips and toes when patients are exposed
to cooler temperatures. To add to this linguistic complexity, pernio can often be
secondary to lupus erythematosus. Lupus pernio should not be confused with lupus
vulgaris, which manifests as painful skin lesions of tuberculosis most commonly
.found on the face
Sarcoidosis is a systemic granulomatous disease of unknown etiology that most
commonly involves the lungs, liver, lymph nodes, skin, and eyes. It frequently affects
persons in the third or fourth decade of life but is also seen in children and the elderly.
In the United States, the incidence and severity of sarcoidosis is higher in African
.Americans
The etiology is unknown, but the pathogenesis appears to be upregulation of the
CD4+ T-helper cells of the Th1 phenotype, with increased production of interleukin-2
and interferon-g in various tissue sites, such as in the lungs or skin. Macrophages with
enhanced ability to present antigen are found in increased numbers. The
compartmentalization of T-helper cells and macrophages in the peripheral tissues
leads to lymphopenia and impaired delayed-type hypersensitivity. Research suggests
that sarcoidosis is caused by an exaggerated response to an unknown antigen.
.Infectious causes, such as tuberculosis, have been proposed but not proven
Cutaneous lesions are seen in approximately one third of sarcoidosis patients,
although some sources report a much higher percentage. The cutaneous lesions can be
divided into sarcoid-specific lesions and nonspecific lesions. The most common
sarcoid-specific lesions are reddish-brown or purple papules and plaques. With
diascopy, lesion color turns to that of apple jelly. However, sarcoidosis can be a great
imitator, and less common manifestations of cutaneous sarcoidosis include
hypopigmented lesions, subcutaneous nodules, ulcerations, ichthyosis, and
erythroderma. It is common to find sarcoid lesions in areas of previous trauma, scars,
and even tattoos. Erythema nodosum is a nonspecific cutaneous finding often seen in
.the setting of sarcoidosis
As noted in our patient's biopsy, sarcoid-specific skin lesions show superficial and
deep dermal epithelioid granulomas with minimal inflammation (naked granulomas).
Multinucleated giant cells of the Langerhans type are often present, and asteroid
.and/or Schaumann bodies can be seen
Hypercalcemia is detected in about 10% of patients and is due to increased levels of
1,25-dihydroxyvitamin D3 produced by the macrophages in the granulomas and
subsequent increased calcium absorption. Other abnormal laboratory findings include

lymphopenia, elevated erythrocyte sedimentation rate, hyperglobulinemia, and


.increased angiotensin-converting enzyme levels
Treatment of sarcoid-related skin lesions can be difficult. If there is systemic
involvement, the treatment of choice is systemic corticosteroids. The skin lesions will
often improve with the corticosteroids, but when the dose is tapered, the skin lesions
can recur. If only the skin is involved, topical or intralesional steroids may be
considered. Antimalarials such as hydroxychloroquine and chloroquine have been
used, as well as methotrexate, thalidomide, and minocycline. Recent reports have
.documented some success with biologic immunomodulators, such as infliximab
Distinguishing dysplastic nevi
CASE NUMBER 1

Dysplastic nevus with mild atypia


The dysplastic nevus initially was described in the 1980s as a mole with increased risk
of developing melanoma and requiring total removal. However, dermatopathologists
now consider dysplastic nevi to be merely one type of benign skin mole, namely the
Clark nevus. The chances a dysplastic nevus will become malignant are no greater
.than those of any other benign nevus, but it can happen, so all moles bear watching
The common mole is made up of pigmented nevus cells. Most people are born
without any moles but will have approximately 40 moles by age 40. Moles often
appear to increase in size and number during the years of increased hormonal activity
.(such as adolescence and pregnancy and when starting oral contraceptives)
Moles are not necessarily all bad. In insects, melanin is the primary immune response
mechanism. Humans have a more sophisticated defense system that involves
antibodies and the like, but we retain our primordial immunologic forms as well.
Melanocytes are not simply pigment-producing cells; they manufacture substances
with a range of biological functions, including structural strengthening by crosslinking proteins, antimicrobial defense, photon shielding, and chemoprotection.
Melanocytes and moles provide several physiologically significant functions,
including communication links with several different systems, e.g., the skin, central
nervous system, and the immune/inflammatory response.1,2
As they apply to nevi, the words dysplastic and atypia continue to confuse
patients, primary-care physicians, and dermatologists when it comes to definition and
significance. The loose use of the term dysplastic in our present histologic
melanocyte terminology is, at best, misleading. Many dermatologists, including
myself, consider dysplastic nevi to be a form of benign acquired melanocytic nevus.
In fact, the NIH has, since 1992, recommended eliminating the terms dysplastic and
atypia in the discussion of nevi. Instead the terminology should be in line with other

pathologic fields in which dysplastic indicates abnormal growth or development as


.well as implying that the lesion is a direct precursor of malignancy

Some authors believe that all nevi should be considered precursors of melanoma. But
if a progressive transformation exists from normal melanocyte to melanoma, the point
along the continuum at which a mole should always be excised prophylactically needs
.to be defined
Nuclear atypia is not a separate criterion for dysplastic nevi but rather can be seen
with any melanocytic nevi. Many additional surgical excisions and repeat excisions
continue to be performed unnecessarily because of an improper understanding of
.dysplastic nevi
Dysplastic nevi should be considered benign and treated accordingly, with repeated
excisions reserved for only those showing clinically significant cytologic atypia (just
as with any other type of nevus).3,4 In short, a coherent, sensible classification of
melanocytic nevi should be established. Despite pleas for a panel of
dermatopathologists to expound the truth about this confusing terminology,5 all they
have provided thus far is a survey on the confusion related to this entity.6

Because the terminology still causes confusion, I like to obtain patients' records from
previous physicians. According to those records, two of the moles previously removed
were read as dysplastic nevus with mild atypia; one was a compound nevus. The two
moles I removed were also read as dysplastic nevus with mild atypia. Although the
pathologist stated that the lesion extended beyond the lateral margins of the biopsy, it
was benign and did not need further excision. However, because of the clinical
appearance of a few other moles with various hues, I will be
monitoring this patient every six months as well as having her
.continue to inventory her moles at home
Distinguishing dysplastic nevi
CASE NUMBER 2

Dysplastic nevus with severe dysplasia


The histology of our second case revealed dysplastic nevus with severe atypia. The
presence of nevus cells on the lateral margin of a dysplastic nevus with severe
dysplasia requires wider excision. In addition to that nevus, four other abnormalappearing moles were excised at the patient's return visit. The pathologist reported
two dysplastic nevi with moderate atypia and two with severe atypia. Given the
clinical picture, I have asked a surgeon to remove all nevi that are >10 mm in
diameter and those that are a mixture of tan, brown, and pink. I also will follow the

.patient in my office
With respect to our two cases, there was not much clinical difference. No one feature
or test declares that a mole has turned malignant, and the clinical features suggesting
abnormality are not always obvious. Four warning signs include A for asymmetry, B
for border (smudgy, ill-defined borders may spell trouble), C for color (variegated
colors, dark brownish-black hues, as well as red or white areas within a mole make it
suspicious), and D for diameter (if >0.6 cm). Also, pigment cells within a mole that
.seem to be overactive, such as a dark dot or streak, may suggest a need for biopsy
It's a good idea to take photos (or video) so you can look closely for changes in moles.
Crusting, erosion, oozing, or bleeding are also signs of possible trouble. Congenital
nevi that are >1.5 cm or very dark pose a risk of developing melanoma. Development
.of any new moles after age 40 are slightly suspicious by history alone
Currently, the gold standard for diagnosing melanocytic neoplasms is by
histopathologic examination. Although dermatopathologists note that dysplastic nevi
cannot be considered a distinct clinicopathologic entity because histologic dysplasia is
found in a range of nevi that may or may not show clinical atypia,7 the pathologist
.who reads the slide has the final word in interpreting a mole's tumor potential

Almost all dermatopathologists presently label dysplastic nevi by degrees of mild,


moderate, or severe. The grading is done on the mole's cytology and architecture. If
read as moderate or severe by either parameter, total removal of the mole is
.recommended. Most pathologists state the need for further removal in the final report
Given the lack of specific criteria for histologic reading of this entity and the variation
among pathologists on mole terminology, it is best to confer directly with the
.physician who read the slides if there is any confusion
In defense of pathologists, lawyers and the lay public often fail to accept that a level
of uncertainty exists with the reading of pathologic slides. Thus, underreading of mole
severity can lead to litigation based solely on poor clinical outcome. As a result, there
will always be a tendency to overread. In other words, a pathologist who follows
common and accepted methods of reviewing, interpreting, and reporting the findings
and who fulfills the standard of care in specimen processing may still be safer to
overread. This is especially true with dysplastic nevi in which no firm criteria
clearly classify moles as benign or malignant. Pathologists would rather speak of a
continuum or spectrum of melanocytic neoplasia and the challenges inherent in
.microscopic interpretation
Thus, neither of these two clinically worrisome moles proved to be melanoma
histologically. With melanoma, the key to success is prompt removal before the lesion

spreads to internal organs, so it behooves the physician treating a suspicious lesion to


know what each pathologist means by atypical nevi and the
.difference between mild and severe dysplasia
Enlarging purple pruritic nodules on the back of the neck

Keloid scars following zoster

The nodules were keloid scars due to herpes zoster. With age-related decrease in cellmediated immunity, the varicella-zoster virus (VZV) that has been lying dormant in a
dorsal root ganglion (likely since childhood) can reactivate and cause the eruption of
.zoster, otherwise known as shingles
Since virus reactivation often occurs in just one ganglion, the eruption follows a
single dermatome. There is frequently a prodromal period lasting a few days during
which the patient experiences burning, itching, or pain. For some patients, this
dysesthesia is the extent of their disease, and no visible lesions appear. For most,
though, the prodrome is followed by an eruption that can begin as urticarial red
papules but quickly becomes vesicular. Vesicles go from clear to turbid before
crusting over and resolving. Pain, a hallmark of zoster, can remain long after the
.vesicles have crusted over and resolved
Besides age, other causes of diminished cell-mediated immunity can result in
reactivation of VZV. Chemotherapy, malignancy, and HIV disease are potential
triggers. Zoster is approximately 15 times more common in HIV-affected individuals
than in their healthy counterparts. In fact, absent another explanation, HIV testing is
warranted in any patient whose zoster appears at a young age or or has an atypical
presentation, such as disseminated zoster. Zoster is approximately four times more
.common in Caucasians but affects all races
A number of complications are associated with zoster. The most common is
postherpetic neuralgia (PHN). In PHN, the pain associated with the shingles eruption
lasts long after clearance of the visible rash. The risk of PHN increases sharply with
age and is rare in patients younger than 50 years. The burning pain can be so severe

that it may begin to interfere with day-to-day functioning. While the pain eventually
.regresses, it can last for up to a year before resolving
It is better to prevent PHN than to treat it. Starting anticonvulsants, such as gabapentin
(Neurontin), at the same time as antivirals in someone who has significant pain and is
at high risk for PHN due to age can help to prevent long-lasting neuropathic pain. For
established PHN, topical agents, such as lidocaine patches or capsaicin, can be used
along with systemic agents, such as tricyclic antidepressants and anticonvulsants. In
my experience, lidocaine patches used topically while titrating the proper dose of
.gabapentin are most effective
There are many other complications. Herpes zoster ophthalmicus involves the
ophthalmic division of the trigeminal nerve. It should be suspected in any patient with
cutaneous lesions in the same distribution, including the forehead or tip of the nose,
and necessitates referral to an ophthalmologist for slit-lamp examination. Involvement
of other cranial nerves can lead to facial nerve palsies that resemble Bell's palsy.
Disseminated lesions occurring across multiple dermatomes may signal a viremia and
.herald visceral involvement, which can be life-threatening
Our patient had a very unusual complication of his zosterkeloids in the areas of his
eruption. Keloids or hypertrophic scars are the result of an exaggerated woundhealing response. Thick scar tissue is laid down following a relatively minor injury. In
hypertrophic scars, the tissue is limited to the area of trauma, whereas in keloids, the
scar tissue extends beyond the initial boundaries of injury. Keloids are much more
common in darker-skinned individuals and occur most often in areas of tension, such
.as the face, neck, chest, and back
Corticosteroids are the mainstay of standard treatment. Intralesional triamcinolone is
more effective than topical treatment. For topical treatment, occlusion, such as with
flurandrenolide tape, is more effective than creams. More persistent lesions can be
treated with intralesional 5-fluorouracil, radiation, and cryosurgery. While excision of
keloids with primary closure can be used, the risk for recurrence is high. To mitigate
this risk, imiquimod (Aldara) and silicone sheeting should be applied topically while
.triamcinolone is injected intralesionally into the healing wound
The young age of our patient at onset of zoster is a reflection of his underlying HIV
disease. Even if he did not have a known HIV diagnosis, testing would have been
indicated based on his presentation. His keloid formation is likely due to the fact that
he is African American and had zoster over the neck. Most interesting, while there
have not been many reports of keloids complicating zoster, several of the patients
involved were also HIV-positive. HIV testing is likely warranted in all patients with
such scarring after shingles. Since our patient's zoster had already resolved, he was
not treated with antivirals. His keloids were injected with triamcinolone 20 mg/cc
.each month, with good results

Enlarging vesicular peeling of the palms


Diagnosis: Keratolysis exfoliativa

The patient was diagnosed with keratolysis exfoliativa, a relatively common,


asymptomatic, noninflammatory, idiopathic keratolytic disorder. Peeling usually
occurs on the palmar surface of the hands and fingers, although it may also appear on
the plantar surface of the feet. Infrequently, the dorsum of the hands, the dorsum of
the feet, or both are involved. The condition can be recurrent, and each episode can
last from several days to a week or longer.1 Recently, some investigators have
recommended that the disorder be referred to as recurrent focal palmar peeling.2
Diagnosis is established by the initial morphology of the skin lesions, the evolution of
the lesions from superficial macules to empty vesicles to desquamative patches, and
.a history of oral medication use for an antecedent upper respiratory tract infection
The etiology of the condition is unknown. Neither the occurrence of dyshidrotic
dermatitis nor the presence of tinea infection has been definitively associated with its
development.1 However, as happened in this case, the temporal association of a prior
infection of the upper respiratory tract with or without the intake of oral
medication has been described in other keratolysis exfoliativa patients.1

Keratolysis exfoliativa
begins as white macules, which are often symmetrically located on the palms and
range in size from <1 to 4 mm in diameter. The uppermost layers of the skin become
separated from the underlying layers of epidermis to form an empty vesicle; if the
lesion is punctured, no fluid is found. The lesion has also been called a corneal layer
vesicle, a dry vesicle, and an air bubble. As the vesicle enlarges, it subsequently
breaks in the center. Peeling occurs, and collarettes of paper-thin scales form at the
edges of the lesions; at this stage, the lesions clinically resemble superficial flaccid
vesicles that have recently ruptured. Eventually, most of the palmar surface is affected
.as older lesions coalesce and new lesions appear
The differential diagnosis includes (1) contact dermatitis, typically characterized by
pruritus, fluid-filled vesicles, and positive patch testing if the etiology is secondary to
an allergic agent, (2) dyshidrotic dermatitis, in which, in addition to pruritic lesions

and vesiculation, there is frequently a chronic history of hyperhidrosis, (3)


dermatophytosis, a superficial infection for which the causative fungal organism can
usually be demonstrated in a KOH preparation or fungal culture taken from the lesion,
(4) postinflammatory desquamation, which often shows more extensive and
prolonged exfoliation, such as that seen in some patients with scarlet fever, and (5)
secondary syphilis, in which the palmar and plantar lesions are associated with a
positive serologic examination for spirochete infection.1,2
Keratolysis exfoliativa is a self-limited condition. Topical treatment with an emollient
cream can be helpful for managing the dryness and for expediting resolution of the
scaling. Often, the cream that is used contains a keratolytic agent, such as a-hydroxy
acid, salicylic acid, tretinoin, or urea.1
Our patient was advised to start twice-daily topical treatment with an OTC emollient
(Eucerin cream). Prompt improvement, manifested as diminished peripheral scaling,
was seen within two days after beginning treatment. The peeling resolved completely
.after one week of therapy
Erythematous truncal plaques
CASE #1: Tinea incognito

The atypical lesions referred to as tinea incognito are the result of morphologic
changes induced by topical corticosteroid treatment of tinea corporis. The changes
may include loss of the raised, scaly, advancing border that is characteristic of tinea
.corporis lesions and a more widespread eruption
A superficial dermatophyte infection, tinea corporis can involve the skin of the trunk
and extremities, while excluding the scalp, beard area, face, hands, feet, and groin.
Commonly seen in tropical regions, tinea corporis can be found worldwide. Its most
common global cause is Trichophyton rubrum, followed by Trichophyton
mentagrophytes; however, tinea corporis can be caused by any of the dermatophytes.
The organisms are transmitted from human to human, animal to human, or soil to
human. Domestic animals are common carriers of the zoophilic dermatophyte
species.1

Tinea corporis is most


commonly characterized by one or more circular, slightly scaly, erythematous plaques
with central clearing and a prominent advancing edge, creating an annular outline and
leading to the lay term ringworm. These lesions are generally associated with
pruritus or burning. Widespread lesions of tinea corporis can occur in
immunodeficient states and may be the presenting sign of AIDS.2
Tinea corporis can mimic many dermatoses, including nummular eczema, contact
dermatitis, psoriasis, granuloma annulare, parapsoriasis, and pityriasis rosea. The
atypical appearance of tinea incognito is more likely than classic tinea corporis to be
confused with other entities, and not uncommonly, biopsy of a chronic refractory
dermatosis will reveal tinea incognito.2
Classic lesions of tinea corporis can be easily recognized clinically; however, the
diagnosis can be easily confirmed by visualizing fungal elements microscopically in
skin scrapings.2 Microscopic examination is performed by scraping scale from the
lesion onto a glass slide with either a scalpel blade or the side of another glass slide. A
10% KOH solution is added to the scrapings to dissolve cellular components while
leaving fungal hyphae intact. This process can be accelerated by gently heating the
slide with a flame. The slide is covered with a cover slip and viewed under the
microscope. The presence of fungal hyphae is considered a positive result.
Additionally, tinea corporis and tinea incognito may be diagnosed with fungal culture
.or skin biopsy using appropriate fungal stains
Localized lesions of tinea corporis can be treated with topical antifungals once or
twice daily for at least four weeks and for at least one week after symptoms resolve.
Topical antifungal therapies include terbinafine, ketoconazole, miconazole,
clotrimazole, tolnaftate, ciclopirox, naftifine, econazole, oxiconazole, butenafine, or
sulconazole.3 Oral terbinafine, fluconazole, or itraconazole may be required for
recalcitrant or extensive lesions.1

In our patient with recalcitrant lesions, oral terbinafine 250 mg daily for four weeks
resulted in complete resolution of symptoms. No scaly lesions were noted at the
.follow-up examination, and the patient did not complain of persistent pruritus
Erythematous truncal plaques
CASE #2: Psoriasis

A very common skin condition, psoriasis affects approximately 2% of the world's


population. The disease can occur at any age, but 75% of cases occur before age 40
years. Frequently, there is a genetic predisposition to psoriasis, with many individuals
reporting a positive family history.4
External factors are important in the pathogenesis of psoriasis. The isomorphic or
Koebner phenomenon describes the appearance of lesions two to six weeks after an
insult to the skin, such as trauma, sunburn, morbilliform drug eruption, or viral
exanthem.4 This phenomenon may also explain the affinity of psoriatic lesions for
.chronically traumatized areas of the body, such as the elbows, knees, and presacrum

Systemic factors are


also important in triggering and affecting certain types of psoriasis. Streptococcal
infections may cause a flare of guttate psoriasis. More severe disease may be seen in
HIV-infected patients. Pregnancy may exacerbate or relieve disease. Multiple
medications can exacerbate psoriasis, including lithium, interferon, beta blockers, and
antimalarials, as can rapid corticosteroid tapers.4 Given the potential for posttreatment disease flare, systemic corticosteroids are generally avoided in patients with
.psoriasis

Recent reports indicate that psoriatic patients have increased risk of a number of
comorbidities. Individuals with more widespread psoriasis have a higher incidence of
cardiovascular disease, hypertension, obesity, and psoriatic arthritis. Moreover, these
patients are at a higher risk of mortality and are more likely than the general
population to smoke and drink alcohol to excess.5 Both dermatologists and primarycare clinicians must be prepared to screen for and address these issues in the psoriatic
.population
A spectrum of clinical features is seen in psoriasis, and psoriatic lesions may have
different morphologies depending on where the lesions are located. Plaque-type
psoriasis usually presents as sharply defined erythematous plaques with overlying
silvery scales most commonly on the elbows, knees, presacrum, and scalp. Lesions
are also frequently found on the hands, feet, and genitals; lesions may also be seen,
albeit less often, anywhere on the body.4 Plaques may be asymptomatic or associated
with pruritus or a burning sensation.6 Removal of the scale on a psoriatic plaque may
lead to pinpoint bleeding; this phenomenon is called Auspitz sign.4
The differential diagnosis of psoriasis includes eczema, lupus erythematosus,
dermatomyositis, seborrheic dermatitis, pityriasis rosea, lichen planus, psoriasiform
syphilis,6 squamous cell carcinoma in situ, cutaneous T-cell lymphoma,4 and tinea
corporis. Psoriasis can often be diagnosed clinically; confirmation can be made
readily by histologic examination of skin biopsy specimens revealing confluent
parakeratosis and regular acanthosis of the epidermis with suprapapillary thinning of
the epidermis, dilated capillaries in the dermal papillae, and neutrophils in the stratum
corneum and spinous layer of the epidermis.4
The available treatments for psoriasis are extensive and, therefore, will not be covered
in detail here. Therapy for the psoriatic patient is based on both extent of disease and
response to previous therapies. Patient comorbidities and convenience of obtaining
.and administering various therapies should also be considered
Limited plaque-type psoriasis can generally be managed with topical therapy. The
most commonly utilized topical therapies are corticosteroids and vitamin D analogs,
such as calcipotriene. Long-term use of topical steroids can lead to local adverse
effects, so these agents must be prescribed judiciously.6 Phototherapy remains a costeffective treatment with relatively limited side effects for extensive disease, but
patients must return for treatment multiple times each week. Numerous systemic
agents, such as methotrexate and cyclosporine, have been available for some time for
treatment of more severe or recalcitrant disease and are still widely used today.6
.These treatments carry potentially serious risks and require close monitoring
Also available for the treatment of psoriasis are newer biologic immunomodulators,
including tumor necrosis factor (TNF)-alpha antagonists and agents that inhibit T-cell
activation.7 These agents have demonstrated dramatic results in many psoriatic
patients; however, they are expensive and have potentially serious side effects,
including an increased risk of infection or reactivation of latent infections, such as
TB.7 One biologic medication, efalizumab, was pulled from the market because of

increased risk of progressive multifocal leukoencephalopathy. The long-term risks


.associated with use of biologic medications are not entirely clear
Our patient was given adalimumab, a monoclonal antibody to TNF-alpha, because he
was not responding to topical therapies and lived too far away for phototherapy to be
feasible. Three months after initiating therapy, he had almost complete resolution of
.the psoriasis, with only a small residual plaque on his left elbow

Potrebbero piacerti anche